You are on page 1of 73

JEE (MAIN+ADVANCED)

PROBABILITY
JEE (MAIN+ADVANCED)

PROBABILITY
CONTENT
S.No Pages

1. Theory 01 – 29

2. Exercise-1 (Special DPP) 30 – 45

3. Exercise-2 46 – 47

4. Exercise-3 (Section-A) 48 – 52
[Previous years JEE-Advanced problems]

5. Exercise-3 (Section-B) 53 – 55
[Previous years JEE-Main problems]

6. Exercise-4 (Section-A) 56 – 59
[Previous years CBSE problems]

7. Exercise-4 (Section-B) 60 – 61
[Potential Problems for Board Preparations]

8. Exercise-5 (Rank Booster) 62 – 63

9. Answer Key 64 – 67
PROBABILITY

PROBABILITY

RANDOM EXPERIMENTS:
In our dayto daylife, we perform manyactivities which have a fixed result no matter anynumber of times
they are repeated. For example given any triangle, without knowing the three angles, we can definitely
say that the sum of measure of angles is 180°.
We also perform many experimental activities, where the result maynot be same, when theyare repeated
under identical conditions. For example, when a coin is tossed it may turn up a head or a tail, but we are
not sure which one of these results will actually be obtained. Such experiments are called random
experiments.
An experiment is called random experiment if it satisfies the following to conditions :
(i) It has more than one possible outcome.
(ii) It is not possible to predict the outcome in advance.
Example :
(i) Tossing a coin is a random experiment.
(ii) Throwing a dice is a random experiment.
(iii) Drawing a card from a well shuffled deck of 52 playing card is also a random experiment.

OUT COMES AND SAMPLE SPACE :


A possible result of a random experiment is called its outcome.
Consider the experiment of rolling a die. The outcomes of this experiment are 1, 2, 3, 4, 5 or 6, if we are
interested in the number of dots on the upper face of the die.
The set of outcomes {1, 2, 3, 4, 5, 6} is called the sample space of the experiment.
Thus, the set of all possible outcomes of a random experiment is called the sample space associated
with the experiment. Sample space is denoted by the symbol S.
Each element of the sample space is called a sample point. In other words, each outcome of the
random experiment is also called sample point.

Illustration :
Two coins (a one rupee coin and a two rupee coin) are tossed once. Find a sample space.
Sol. Clearly the coins are distinguishable in the sense that we can speak of the first coin and the
second coin. Since either coin can turn up Head (H) or Tail (T), the possible outcomes may be
Heads on both coins = (H, H) = HH
Head on first coin and Tail on the other = (H, T) = HT
Tail on first coin and Head on the other = (T, H) = TH
Tail on both coins = (T, T) = TT
Thus, the sample space is S = {HH, HT, TH, TT}

Illustration :
When a coin is tossed twice if head appears in the second throw then a dice is thrown. Write down
the sample space of the experiment.
Sol. When a coin is tossed two times then possible outcomes are {(TT), (HT), (TH), (HH)}
If head appears in the second throw then dice is thrown.
All possible outcomes of the experiment are
S = {(TT), (HT), (TH1), (TH2),(TH3),(TH4),(TH5), (TH6), (HH1),(HH2), (HH3), (HH4), (HH5), (HH6)}

BANSAL CLASSES Private Ltd. ‘Bansal Tower’, A-10, Road No.-1, I.P.I.A., Kota-05 Page # 1
PROBABILITY

EVENT :
Consider the experiment of tossing a coin two times.An associated sample space is
S = {HH, HT, TH, TT}
Now suppose that we are intersected in those outcomes which correspond to the occurrence of exactly
one head. We find that HT and TH are the only elements of S corresponding to the occurrence of this
happening (event). These two elements form the set E = {HT, TH}.
We know that the set E is a subset of the sample space S. Similarly, we find the following correspondence
between events and subsets of S.
Description of events Corresponding subset of 'S'
Number of tail is exactly 2 A = {TT}
Number of tails is atleast one B = {HT, TH, TT}
Number of heads is atmost one C = {HT, TH, TT}
Second toss is not head D = {HT, TT}
Number of tails is atmost two S = {HH, HT, TH, TT}
Number of tails is more than two 

Definition :
Any subset E of a sample space S is called an event.
Note : The maximum number of events which can be associated with an experiment is 2n, where n is the number
of elements in the sample space.
i.e., nC0 + nC1 + nC2 + .... + nCn = 2n

Illustration :
In throwing a pair of dice write down two possible events.
E1 = sum of the numbers appear on both the dice is 7.
E2 = The sum of the numbers appear on both the dice is divisible by 3.
Sol. E1 = {(6, 1), (5, 2), (4, 3), (3, 4), (2, 5), (1, 6)}
E2 = {(2, 1), (1, 2), (5, 1), (4, 2), (3, 3), (2, 4) (1, 5), (6, 3), (5, 4), (4,5), (3, 6), (6, 6)}

Occurrence of an event :
Consider the experiment of throwing a die. Let E denotes the event " a number less than 4 appears". If
actually '1' had appeared on the die then we say that event E has occurred. As a matter of fact if
outcomes are 2 or 3, we say that event E has occurred.
Thus, the event E of a sample space S is said to have occurred if the outcome  of the experiment is such
that  E. If the outcome  is such that  E, we say that the event E has not occurred.

Impossible and Sure Events :


The empty set  and the sample space S describe events. In fact  is called an impossible event and S,
i.e., the whole sample space is called the sure event.
To understand these let us consider the experiment of rolling a die. The associated sample space is
S = {1, 2, 3, 4, 5, 6}
Let E be the event" the number appears on the die is a multiple of 7".
Clearly no outcome satisfies the condition given in the event, i.e., no element of the sample space ensure
the occurrence of the event E. Thus, we say that the empty set only correspond to the event E. In other
words we can say that it is impossible to have a multiple of 7 on the upper face of the die. Thus, the event
E =  is an impossible event.
Now let us take up another event F " the number turns up is odd or even". Clearly F = {1, 2, 3, 4, 5, 6}= S,
i.e., all outcomes of the experiment ensure the occurrence of the event F. Thus, the event F = S is a sure
event.
BANSAL CLASSES Private Ltd. ‘Bansal Tower’, A-10, Road No.-1, I.P.I.A., Kota-05 Page # 2
PROBABILITY

Simple Event :
If an event E has only one sample point of a sample space, it is called a simple (or elementary) event.
In a sample space containing n distinct elements, there are exactly n simple events.
S = {HH, HT, TH, TT}
There are four simple events corresponding of this sample space. These are
E1 = {HH}, E2 = {HT}, E3 = {TH} and E4 = {TT}.

Compound Event :
If an event has more than one sample point, it is called a compound event.
For example, in the experiment of "tossing a coin thrice" the events
E : 'Exactly one head appeared'
F : ' Atleast one head appeared'
G : 'Atmost one head appeared' etc.
are all compound events. The subsets of associated with these events are
E = {HTT, THT, TTH}
F = {HTT, THT, TTH, HHT, HTH, THH, HHH}
G = {TTT, THT, HTT, TTH}
Each of the above subsets contain more than one sample point, hence they are all compound events.

ALGEBRA OF EVENTS :
In the Chapter on Sets, we have studied about different ways of combining two or more sets, viz, union,
intersection, difference, complement of a set etc. Like-wise we can combine two or more events by
using the analogous set notations.
Let A, B, C be events associated with an experiment whose sample space is S.

Complementary Event :
For every eventA, there corresponds another event A' or A called the complementary event to A. It is
also called the event 'not A'.
For example, take the experiment 'of tossing three coins'.An associated sample space is
S = {HHH, HHT, HTH, THH, HTT, THT, TTH, TTT}
Let A = {HTH, HHT, THH} be the event 'only one tail appears'.

Clearly for the outcome HTT, the event A has not


occurred. But we may say that the event 'not A' has
occurred. Thus, with every outcome which is not inA,
we say that 'not A' occurs. A

Thus the complementary event 'not A' to the event Ais


A' = {HHH, HTT, THT, TTH, TTT} Aor A
or A' = { :  S and  A} = S – A

BANSAL CLASSES Private Ltd. ‘Bansal Tower’, A-10, Road No.-1, I.P.I.A., Kota-05 Page # 3
PROBABILITY

The Event 'A or B' :


Recall that union of two sets Aand B denoted byA B contains all those elements which are either in
A or in B or in both. A B S
When the sets A and B are two events associated with
a sample space, then 'A  B' is the event 'eitherA or B
or both'. This event 'A  B' is also called 'A or B'.

Therefore Event 'A or B' = A  B = { :  A or  B} AB

The Event 'A and B' :


We know that intersection of two sets A  B is the set of those elements which are common to both A
and B. i.e., which belong to both 'A and B'.
If A and B are two events, then the set A  B denotes the event 'A and B'.
Thus, A  B = { :  A and  B}
For example, in the experiment of 'throwing a die twice'
A B
Let Abe the event 'score on the first throw is six' and B
is the event 'sum of two scores is atleast 11' then
A = {(6, 1), (6, 2), (6, 3), (6, 4), (6, 5) (6, 6)}
and B = {(5, 6), (6, 5), (6, 6)}
so A  B = {(6, 5), (6, 6)} AB
Note that the set A B = {(6, 5), (6, 6)} may represent the event ' the score on the first throw is six and
the sum of the scores is atleast 11.'

The Event 'A but not B' : A B


We know that A – B is the set of all those elements
which are in A but not in B. Therefore, the set A – B
may denote the event 'A but not B'. We know that
A – B = A  B' AB

The Event 'neither A nor B' : A B


The set of the elements which are neither in setAnor in
set B. i.e. S – (A  B) and which is denoted on A  B .
AB
Illustration :
Consider the experiment of rolling a die. Let A be the event 'getting a prime number', B be the
event 'getting an odd number'. Write the sets representing the events
(i) A or B (ii) A and B (iii) A but not B (iv) 'not A'.
Sol. Here S = {1, 2, 3, 4, 5, 6}, A = {2, 3, 5}
Obviously A B
(i) 'A or B' = A  B = {1, 2, 3, 5}
(ii) 'A and B' = A  B = {3, 5} 2 1
(iii) 'A but not B' = A – B = {2} 4 6
(iv) 'not A' = A' = {1, 4, 6}
Note : ( i ) A  B  A  B 
 De Morgan' s Law
( ii ) A  B  A  B 
BANSAL CLASSES Private Ltd. ‘Bansal Tower’, A-10, Road No.-1, I.P.I.A., Kota-05 Page # 4
PROBABILITY

THREE MOST IMPORTANT EVENTS :

(1) Equally Likely Events :

Events are said to be equally likely when no particular event has preference to occur in relation to the
other event.

Example :
(i) The outcomes as a result of throwing a die are equally likely, as no particular face is more likely to occur
as compared to other faces. That is why we normally write as fair die or unbiased die.
(ii) The outcomes as result of drawing a card from a well shuffled pack of 52 playing cards are equally likely
to occur. Each card is as likely to be withdrawn as any other card.
(iii) How ever getting of a total of 7 is not as equally likely as getting of a total of 12 when a pair of dice are
rolled once. It is also to be noted that it is 6 times more likely to get a total of 7 than to get a total of 12
in a single throw with the pair of dice.

(2) Mutually Exclusive / Disjoint / Incompatible Events :

Two eventsAand B are said to be mutually exclusive A B


events if their simultaneous occurrence is impossible,
AB= 
i.e. both the events can not occur together.

Example :
(i) In throwing a fair die, to events A and B are such that
A : getting an odd number
B : getting an even number
then A& B are mutually exclusive events.

(ii) In drawing a card from a well shuffled pack of 52 playing card two events A and B are such that
A : getting an ace
B : getting a red card
thenAand B are not mutually exclusive events.

(3) Exhaustive Events :

If E1, E2, ...., En are 'n' events associated with an experiment whose sample space is S and if
n
E1  E2  E3  ...... En =  Ei  S
i 1

then E1, E2,......., En are called exhaustive events. In other words, events E1, E2, ......., En are said to be
exhaustive if atleast one of them necessarily occurs, whenever the experiment is performed.
n
Further, if Ei  Ej =  for i  j i.e., events Ei and Ej are pairwise disjoint and  Ei  S , then
i 1

events E1, E2,...., En are called mutually exclusive and exhaustive events.

BANSAL CLASSES Private Ltd. ‘Bansal Tower’, A-10, Road No.-1, I.P.I.A., Kota-05 Page # 5
PROBABILITY

Example :
Consider the experiment of throwing a die. We have
S = {1, 2,3, 4, 5, 6}. Let us define the following events
A : 'a number less than 4 appears'.
B : 'a number greater than 2 but less than 5 appears'
and C : 'a number greater than 4 appears'.
Then A = {1, 2, 3}, B = {3, 4} and C = {5, 6}. We observe that
A  B  C = {1, 2, 3}  {3, 4}  {5, 6} = S.
Such events A, B and C are called exhaustive events.

CLASSICAL (A PRIORI) DEFINITION OF PROBABILITY :


If an experiment results in a total of (m + n) outcomes which are equallylikelyand mutuallyexclusive with
one another and if ‘m’ outcomes are favourable to an event ‘A’ while ‘n’are unfavourable , then the
probability of occurrence of the event ‘A’, denoted by P(A), is defined by
m number of favourable outcomes
=
mn total number of outcomes
m
i.e. P(A) = .
mn
n
Note that P( A ) or P(A) or P(AC), i.e. probability of non-occurrence of A = = 1 – P(A)
mn
In the above we shall denote the number of outcomes favourable to the event A by n(A) and the total
number of out comes in the sample space S by n(S).
n (A)
 P(A) = .
n (S)
If P(A) = 0 Event is impossible
P(A) = 1 Event is sure
P(A) 1 and P(A) 0
Note :
(i) More is the probability of an event, more are the chances of its happening.
(ii) P() = 0 & P(S) = 1 i.e. nothing outside sample space can occur.

Designation of Cards :
Colours : There are two colours. Red & Black
Suits : There are four (4) suits (types).
Each suit contains 13 cards
Pack
of
cards

(26) (26)
Colours
Black Cards Red Cards

(13) (13) (13) (13)


Spade Club Heart Diamond

BANSAL CLASSES Private Ltd. ‘Bansal Tower’, A-10, Road No.-1, I.P.I.A., Kota-05 Page # 6
PROBABILITY

Recognition of Cards :

K Q J A
King Queen Jack Ace

♥ 1 1 1 1

1 1 1 1

♠ 1 1 1 1

♣ 1 1 1 1

4 4 4 4

(i) Face Cards :


Face cards contain 12 cards all of K, Q and J having designed a figure of a person.
i.e., Face cards = 4 + 4 + 4 = 12,

(ii) Honours Cards :


It contains all face cards and also a card marked A.
i.e. Honours cards = (4 + 4 + 4) + 4 = 16 cards.

(iii) Knave Cards :


(10, J, Q) = 4 + 4 + 4 = 12 cards

Illustration :
An old man while dialing a seven digit telephone number, after having dialed the first five digits,
suddenly forgets the last two. But he remembered that the last two digits were different. On this
assumption he randomly dials the last two digits. What is the probability that the correct telephone
number is dialed.
Sol. Note that total number of ways in which the last two digits (different) can be dialed is
10 × 9 = 90. Out of these 90 EL/ME/ and exhaustive outcomes only one of them favours happening
1
of the event "correct telephone is dialed". Hence P(E) = .
90
What the probability would have been if he did not even remember the last two digits were different:
Here n(S) = 10 × 10 = 100
1
Hence P(E) = .
100

Illustration :
Two natural numbers are randomly selected from the set of first 20 natural numbers. Find the
probability that (A) their sum is odd (B) sum is even (C) selected pair is twin prime.
Sol. S = {1, 2, 3, ......, 19, 20}; n(S) = 20C2
100 10
n(A) = 10C1.10C1 = 100 P(A) = = (sum odd one odd and one even)
190 19

BANSAL CLASSES Private Ltd. ‘Bansal Tower’, A-10, Road No.-1, I.P.I.A., Kota-05 Page # 7
PROBABILITY

90 9
n(B) = 10C2 + 10C2 = 2.10C2 = 90 P(B) = =
190 19
(sum even both odd or both even)
4 2
n(C) = {(3, 5), (5, 7), (11, 13), (17, 19)} P(C) = =
190 95
Illustration :
A leap year is selected at random. Find the probability that it has
(A) 53 Sundays (B) 53 Sundays and Mondays (C) 53 Sundays or 53 Mondays
Sol. Leap year means which is divisible by 4 if it not a century year. If it is a century year it must be
divisible by 400 as well. A leap year has 366 days out of this 364 days are consumed for 52 weeks
i.e. 52 times
S, M, T, W, Th, F and Sat. For remaining 2 days of the leap year can begin with SM, MT, TW, W Th.,
Th. F, F Sat and Sat Sun.
2 1 3
P(A) = ; P(B) = ; P(C) =
7 7 7

ODDS IN FAVOUR AND ODDS AGAINST OF AN EVENT :


If an experiment has (m + n) as a total number of outcomes which are equally likely, mutually exclusive
and exhaustive, and if 'm' outcomes are in favour of an event 'A' and n outcomes are not in favour of that
event Ameans n outcomes are in against of event A then we can say –
m No. of outcomes which are in favour of event A
Odds in favour of event A = =
n No. of outcomes which are not in favour of event A
n No. of outcomes which are not in favour of event A
Odds in against of event A = =
m No. of outcomes which are in favour of event A
a
Note : If P(A) = then
b
(i ) odds in favour of event A = a : b – a.
(ii) odds against of event = b – a: a.

Illustration :
5 different marbles are placed in 5 different boxes randomly. Find the odds in favour that exactly
two boxes remain empty. Given each box can hold any number of marbles.
Sol. n(S) = 55; For computing favourable outcomes.
2 boxes which are remain empty, can be selected in 5C2 ways and 5 marbles can be placed in the
 5! 5! 
remaining 3 boxes in groups of 221 or 311 in 3!   = 150 ways
 2! 2! 2! 3! 2! 

5 150 12
 P(E) = C2  5 =
5 25
Hence, odds in favour of event E = 12 : 13 Ans.

BANSAL CLASSES Private Ltd. ‘Bansal Tower’, A-10, Road No.-1, I.P.I.A., Kota-05 Page # 8
PROBABILITY

DEPENDENT AND INDEPENDENT EVENTS :


Independent events – Events A and B are said to be independent if occurrences or non-occurrence of
one does not affect the probability of occurrence or non-occurrence of the other.

(i) Two people holding a normal dice and the other a coin, throw them once, then getting a 6 on normal dice
and getting a head on the coin are the examples of events which are independent.
(ii) From an urn containing 2R, 3G and 4W balls, a ball is drawn its colour is noted, the ball is replaced in the
urn and another ball is drawn. Getting a red and a red ball on both the occasion are the examples of
events which are independent.
(iii) Similar example can be given in playing cards 'getting an ace' and 'an ace' in two successive draws from
a well shuffled pack of 52 cards when the first drawn card is replaced in the pack before the second is
drawn. If it is not replaced, the events become dependent or contingent.

Note : Dependent/Independent events come from two different experiments while mutually exclusive events
come from the same experiment.

ADDITION THEOREM ON PROBABILITY :


If Aand B are two events associated with an experiment then P(AB) is
A B S
called the sum of the probabilities of all the sample points in A  B or
probability of occurrence of atleast one of the events fromAand B and the
expression for P(AB) is called the addition theorem on probability.
From the Venn diagram it is clear that

P(Occurence atleast one  = P(A) + P(B) – P(AB)



of the events from Aand B)  = P(A) + P ( A  B)

P(A or B or both)  = P(B) + (A  B)
  P(A  B) 
or  = P (A  B) + P(A B) + P (A  B)

P(A + B)  = 1 – P ( A  B)
 = 1 – P ( A  B)

P(occurrence of exactly one of the events)  P(A  B)  P( A  B) A B


or 
P(A or B but not both) P(A)  P(B)  P(A  B)
Note :
(i) IfAand B are mutually exclusive events then –
P(A B) = P(A) + P(B) {P(A B) = 0 }
(ii) If A and B are exhaustive events then P(A B) = 1
(iii) P(A B) = 1 – P ( A  B)

BANSAL CLASSES Private Ltd. ‘Bansal Tower’, A-10, Road No.-1, I.P.I.A., Kota-05 Page # 9
PROBABILITY

Illustration :

Two students Anil and Ashima appeared in an examination. The probability that Anil will qualify
the examination is 0.05 and that Ashima will qualify the examination is 0.10. The probability that
both will qualify the examination is 0.02. Find the probability that
(a) Both Anil and Ashima will not qualify the examination
(b) Atleast one of them will not qualify the examination and
(c) Only one of them will qualify the examination.

Sol. Let E and F denote the events that Anil and Ashima will qualify the examination, respectively.
Given that
P(E) = 0.05, P(F) = 0.10 and P(E  F) = 0.02
Then
(a) The event 'both Anil and Ashima will not qualify the examination' may be expressed as E'  F'.
Since, E' is not E, i.e., Anil will not qualify the examination and F' is 'not F, i.e., Ashima will not
qualify the examination.
Also E'  F' = (E  F)' (By Demorgan's Law)
Now P(E  F) = P(E) + P(F) – P(E  F)
or P(E F) = 0.05 + 0.10 – 0.02 = 0.13
Therefore P(E'  F') = P(E  F)' = 1 – P(E  F) = 1 – 0.13 = 0.87
(b) P(atleast one of them will not qualify)
= 1 – P(both of them will qualify)
= 1 – 0.02 = 0.98
(c) The event only one of them will qualify the examination is same as the event either (Anil will
qualify, and Ashima will not qualify) or (Anil will not qualify and Ashima will qualify) i.e., E  F'
or E'  F, where E  F' and E'  F are mutually exclusive.
Therefore, P(only one of them will qualify) = P(E  F' or E'  F)
= P(E  F') + P(E'  F) = P(E) – P(E  F) + P(F) – P(E  F)
= 0.05 – 0.02 + 0.10 – 0.02 = 0.11

CONDITIONAL PROBABILITY :

Let A and B be any two events associated with a random experiment. B


The probability of occurrence of eventAwhen the event B has already A
occurred is called the conditional probabilityofAwhen B is given and is B
A B
denoted as P(A/B). The conditional probability P(A/B) is meaningful
only when P(B)  0, i.e., when B is not an impossible event.

Bydefinition ,
A
P   = Probability of occurrence of event A when the event B as already occurred
B
Number of cases favourable to B which are also favourable to A
=
Number of cases favourable to B
A Number of cases favourable to A  B
 P  =
B Number of cases favourable to B

BANSAL CLASSES Private Ltd. ‘Bansal Tower’, A-10, Road No.-1, I.P.I.A., Kota-05 Page # 10
PROBABILITY

Number of cases favourable to A  B


A Number of cases in the sample space
Also, P  =
B Number of cases favourable to B
Number of cases in the sample space

 A  P(A  B)
 P  = , provided P(B)  0.
B P(B)
Similarly, we have
 B  P(A  B)
P  = , provided P(A)  0.
A P(A)

Illustration :
In a class, 30% of the students failed in Physics, 25% failed in Mathematics and 15% failed in
both Physics and Mathematics. If a student is selected at random failed in Mathematics, find the
probability that he failed in Physics also.
Sol. Let A be the event "failed in Physics" and B be the event "failed in Mathematics". We want to
 A
find P   . It is given that
B
30 25
P(A) = and P(B) =
100 100
15
Also P(A  B) =
100
 A  P( A  B ) 15 / 100 15 3
Therefore P  =   
B P( B ) 25 / 100 25 5
Illustration :
B A
Let A and B be two events such that P(A) = 0.3, P(B) = 0.6 and P   = 0.5. Then P   equals
 A B
(A) 3/4 (B) 5/8 (C) 9/40 (D) 1/4
B
Sol. P(A  B) = P(A) P   = (0.3) (0.5) = 0.15
 A
Now P(A  B) = P(A) + P(B) – P(A  B) = 0.3 + 0.6 – 0.15 = 0.75
 A  P( A  B ) 1  P( A  B ) 1  0.75 0.25 250 5
Also P   = = = =  
B P( B ) 1  P( B ) 1  0 .6 0.4 400 8

BANSAL CLASSES Private Ltd. ‘Bansal Tower’, A-10, Road No.-1, I.P.I.A., Kota-05 Page # 11
PROBABILITY

MULTIPLICATION THEOREM ON PROBABILITY :


Let A and B be two events associated with a sample space S. Clearly, the set A  B denotes the event
that both A and B have occurred. In other words, A  B denotes the simultaneous occurrence of the
events E and F. The event A  B is also written as AB.
We know that the conditional probability of event A given that B has occurred is denoted by P(A | B)
and is given by
P(A  B)
P(A | B) = , P(B)  0
P(B)
From this result, we can write
P(A  B) = P(B) · P(A | B) …(i)
Also, we know that
P(A  B)
P(B | A ) = , P(A)  0
P(A)
P(A  B)
or P(B | A) = (since A  B = B  A)
P(A)
Thus, P(A  B) = P(A) · P(B | A) …(ii)
Combining (i) and (ii), we find that
P(A  B) = P(A) P(B | A) = P(B) P(A | B) provided P(A)  0 and P(B)  0
The above result is known as the multiplication rule of probability.
A B
Note : IfA& B are independent events then P   = P(A) and P   = P(B) and in this case multiplication
B A
theorem P(A  B) = P(A) · P(B).

Theorem-I :
LetAand B be events associated with a random experiment. IfAand B are independent, then show that
the events (i) A , B (ii) A, B (iii) A, B are also independent.
Proof : The events A and B are independent .
 P(A  B) = P(A) P(B) ……(i)
(i) (A  B)  ( A  B) = (A  A )  (B  B) =  B = 
and (A  B)  ( A  B) = (A  A )  B = S  B = B
 The events A  B and A  B are mutually exclusive and their union is B.
 By addition theorem, we have P(B) = P(A  B) + P( A  B) ……(i)
 P( A  B) = P(B) – P(A  B) = P(B) – P(A) P(B)
= 1 P(A)  P(B) = P( A ) P(B) (Using (i))
 P( A B) = P( A ) P(B) i.e., A and B are independent.
(ii) (A  B)  (A  B ) = (A  A)  (B  B ) = A  = 
and (A  B)  (A  B ) = A (B  B ) = A  S = A
 The events A  B andA  B are mutually exclusive and their union is A.
 By addition theorem, we have P(A) = P(A  B) + P(A  B ) ……(i)
 P(A  B ) = P(A) – P(A  B) = P(A) – P(A) P(B)
= P(A) 1 P(B)  = P(A) P( B ) (Using (i))

BANSAL CLASSES Private Ltd. ‘Bansal Tower’, A-10, Road No.-1, I.P.I.A., Kota-05 Page # 12
PROBABILITY

 P(A  B ) = P(A) P( B ) i.e., A and B are independent.


(iii) ( A  B)  ( A  B ) = ( A  A )  (B  B ) = A  = 
and ( A  B)  ( A  B ) = A (B  B ) = A  S = A
 The events A  B and A  (B  B ) are mutually exclusive and their union is A .
 By addition theorem, we have P( A ) = P( A  B) + P( A  B ) ……(i)
 P( A  B ) = P( A ) – P( A  B) = P( A ) – P( A ) P(B)
= P( A ) 1 P(B)  = P( A ) P( B ) (Using (i))
 P( A  B ) = P( A ) P( B ) i.e., A and B are independent.

Illustration :
A die is thrown. If E is the event 'the number appearing is a multiple of 3' and F be the event 'the
number appearing is even' then find whether E and F are independent?
Sol. We know that the sample space is S = {1, 2, 3, 4, 5, 6}
Now E = { 3, 6} , F = { 2, 4, 6} and E  F = {6}
2 1 3 1 1
Then P(E) =  , P(F) = = and P(E  F) =
6 3 6 2 6
Clearly P(E  F) = P(E) · P(F)
Hence E and F are independent events.

Illustration :
Three coins are tossed simultaneously. Consider the event E 'three heads or three tails', F 'at least
two heads' and G 'at most two heads'. Of the pairs (E, F), (E, G) and (F, G), which are independent?
which are dependent?
Sol. The sample space of the experiment is given by
S = {HHH, HHT, HTH, THH, HTT, THT, TTH, TTT}
ClearlyE = {HHH, TTT} , F = {HHH, HHT, HTH, THH}
and G = {HHT, HTH, THH, HTT, THT, TTH, TTT}
Also E  F = {HHH}, E  G = {TTT}, F  G = {HHT, HTH, THH}
2 1 4 1 7
Therefore P(E) = = , P(F) = = , P(G) =
8 4 8 2 8
1 1 3
and P(E  F) = , P(E  G) = , P(F  G) =
8 8 8
1 1 1 1  7
Also P(E) · P(F) =   , P(E) · P(G) =  
4 2 8 4 8 32
1 7 7
and P(F) · P(G) =  
2 8 16
Thus P(E  F) = P(E) · P(F)
P(E  G)  P(E) · P(G) and P(F  G)  P(F) · P(G)
Hence, the events (E and F) are independent, and the events (E and G) and (F and G) are dependent.

BANSAL CLASSES Private Ltd. ‘Bansal Tower’, A-10, Road No.-1, I.P.I.A., Kota-05 Page # 13
PROBABILITY

Illustration :
There are four machines and it is known that exactly two of them are faulty. They are tested one
by one, in a random order till both the faulty machines are identified. The probability that only
two tests are needed is
1 1 1 1
(A) (B) (C) (D)
3 6 2 4
Sol. The procedure ends in first two tests if either both are faulty or both are good. Therefore the
probability is
G F 2 1 1 1 1
= P(G  G) + P(F  F) = P(G) · P   + P(F) · P   = ·  ·  Ans.
G F 4 3 4 3 3

T H R E E E V E N T S A S S O CI A T E D W I T H A N E X P E R I M E N TA L
PERFORMANCE :

A B

ABC AB C BAC

AB C

A CB BCA


CAB

C A B  C

The addition theorem can be extended when three events are associated with the experiment.
If A, B and C are three events then
P(A B C) denotes the sum of probabilities of all the sample points in (A B C) or probability
of occurrence of atleast one of the events.

(i) P(A B C) = P(A) + P(B) + P(C) – P(A B) – P(B C) – P(C A) + P(A B C)

S
A B

(A B C)

BANSAL CLASSES Private Ltd. ‘Bansal Tower’, A-10, Road No.-1, I.P.I.A., Kota-05 Page # 14
PROBABILITY

(ii) P (occurrence of exactly one of the events) =


P(A) + P(B) + P(C) – 2[P(A B) + P(B C) + P(C A) ] + 3P(A B C)

S
A B

( A  B  C )  ( A  B  C )  ( A  B  C)
C

(iii) P (occurrence of exactly two of the events) =


P(A B) + P(B C) + P(C A) – 3P(A B C)

S
A B

( A  B  C )  ( A  B  C)  ( A  B  C)

(iv) P (occurrence of atleast two of the events) =


P(A B) + P(B C) + P(C A) – 2P(A B C)

S
A B
( A  B  C )  ( A  B  C)  ( A  B  C)  ( A  B  C)

Note :
(a) IfA, B, C are three pair wise mutually exclusive  they are mutually exclusive
however ifA, B, C are mutually exclusive   they are pair wise mutually exclusive

ME 
 pair wise ME Pair wise ME  ME

BANSAL CLASSES Private Ltd. ‘Bansal Tower’, A-10, Road No.-1, I.P.I.A., Kota-05 Page # 15
PROBABILITY

(b) However , ifA,B,C are pair wise independent   they are independent. Infact for 3 events A, B and C
to be independent they must be
(i) pair wise (ii) mutuallyindependent , mathematically
P(AB) = P(A) . P(B) ; P(BC) = P(B).P(C) ; P(CA) = P(C).P(A)
and P(ABC) = P(A).P(B).P(C)
for n independent events, the total number of conditions would be
nC + nC + .... + nC = 2n – n – 1
2 3 n

Illustration :
A, B and C are three newspapers from a city. 25% of the population reads A, 20% reads B, 15%
reads C, 12% reads both A and B, 10% reads both B and C, 8% reads both A and C and 6% reads
all the three. Find the percentage of the population who read atleast one of A, B and C.
Sol. We are given that
25 20 15
P(A) = , P(B) = , P(C) =
100 100 100
12 10 8 6
P (A  B) = , P(B  C) = , P(C  A) = and P(A  B  C) =
100 100 100 100
We have to find P(A  B  C). We can use the formula
P(A  B  C) = P(A) + P(B) + P(C) – P(A  B) – P(B  C) – P(C  A) + P(A  B  C)
1 36
= (25 + 20 + 15 – 12 – 10 – 8 + 6) =
100 100
Thus 30% of the people read atleast one of the newspapers.

Illustration :
Let A, B and C be three events such that
p = P (exactly one of A or B) = P(exactly one of B or C) =P(exactly one of C or A)
and P(A, B, C simultaneously) = p2
1
where 0 < p < . Then P (at least one of A, B or C) is equal to
2
2
3p 2p 2 p  3 p2 2 p  3 p2 3 p  2 p2
(A) (B) (C) (D)
2 2 4 4
Sol. Exactly one of A or B means
So P(A) + P(B) – 2P(A  B) = p …(i)
Similarly P (exactly one of B or C)
P(B) + P(B  C) – 2P(B  C) = p …(ii)
and P(C) + P(A) – 2P(C  A) = p …(iii)
Adding equation (i) – (iii), we have
2[P(A) + P(B) + P(C) – P(A  B) – P(B  C) – P(C  A)] = 3p …(iv)
Now P(atleast one A, B or C) is given by [see part (3), theorem 7.2]
P(A  B  C) = P(A) + P(B) + P(C) – P(A  B) – P(B  C) – P(C  A) + P(A  B  C)
3p
= + p2 [Equation (iv) and P(A  B  C) = p2]
2
3 p  2 p2
=
2

BANSAL CLASSES Private Ltd. ‘Bansal Tower’, A-10, Road No.-1, I.P.I.A., Kota-05 Page # 16
PROBABILITY

BINOMIAL PROBABILITY :
Let an experiment has n-independent trials, and each of the trial has two possible outcomes
(i) success (ii)failure
If probability of getting success, P(S) = p and probability getting failure, P(F) = q such that p + q = 1.
Then, P(r successes) = nCr pr qn–r

Proof :
Consider the compound event where r successes are in succession and (n – r) failures are in succession.
 
P SSS  S FFF 
   = PF  (S).P(S) P(S) P(F).P(F) P(F) = Pr . qn–r
     
 r (n  r)  r times ( n  r ) times

n!
But these r successes and (n – r) failures can be arranged in = nCr ways and in each
r !(n  r )!
arrangement the probability will be pr . qn–r
Hence total pr. = P(r) = nCr pr qn–r ……(1)
Recurrence relation
p(r + 1) = nCr+1 pr+1 . qn–r–1
P(r  1) n C r 1 p n  r p
  n 
P(r ) Cr q r  1 1  p

nr p
 P(r + 1) = · P( r ) ……(2)
r 1 1 p
Equation (2) is used for completely the probabilities of P(1); P(2); P(3); …… etc. once P(0) is
determined.

Illustration :
A pair of dice is thrown 6 times, getting a doublet is considered a success. Compute the probability of
(i) no success (ii) exactly one success
(iii) at least one success (iv) at most one success
Sol. Total sample spaces are = 36
In which six doublets then
3 1 1 5
p= = ; q=1– =
36 6 6 6
(i) No success for r = 0
0 6 6
1 5 5
 p(0) = 0     =  
6C
6  6  6 
(ii) Exactly one success for r = 1
1 5 5
1 5 5
 p(1) = 6C  
1 6
  = 
  6  6

BANSAL CLASSES Private Ltd. ‘Bansal Tower’, A-10, Road No.-1, I.P.I.A., Kota-05 Page # 17
PROBABILITY

(iii) For at least one success for r = 1, 2, 3, 4, 5, 6.


6 5 2 4 3 3 4 2
  6
Cr p r q6 r = 6 C1  1  5  + 6 C2  1   5  + 6 C3  1  5 6 1 5
  + C4    
r 1  6  6  6  6  6  6  6  6 
5 1 6 0
1 5 1 5
+ C5     + 6 C6    
6
6  6  6  6 
(iv) For at most one success for r = 0, 1
1 6 6 r 0 6 5
1 5 1  5  6  1  5 
 6
Cr    
6  6 
6
= C0     + C1   
r 0 6  6   6  6 

TOTAL PROBABILITY THEOREM :


Let E1, E2, ……, En be n mutually exclusive and exhaustive events, with non-zero probabilities, of a
random experiment. If A be any arbitrary event of the sample space of the above random experiment
with P(A) > 0, then
A A A
P(A) = P(E1) P   + P(E2) P   + …… + P(En) P   .
 E1   E2   En 

Proof : Let S be the sample space of the random experiment.


Since E1, E2, ……, En are exhaustive, we have S = E1  E2  ……  En .
Now A = S  A = (E1  E2  ……  En)  A
 A = (E1  A)  (E2  A)  ……  (En  A) ……(i)
Since E1, E2, ……, En are mutually exclusive, we have Ei  Ej =  for i  j
Now (Ei  A)  (Ej  A) = (Ei  Ej)  A =  A = 
 E1  A, E2  A, ……, En  A are also mutually exclusive.
By using addition theorem, (i) implies
P(A) = P(E1  A) + P(E2  A) + …… + P(En  A)
A A A
 P(A) = P(E1) P   + P(E2) P   + …… + P(En) P   .
 E1   E2   En 

Remark : In practical problems, it is found convenient to write as follows :


P(A) = P(E1A or E2A or …… EnA) = P(E1A) + P(E2A) + …… + P(EnA)
A A
P(A) = P(E1) P   + P(E2)   .
 E1   E2 

BANSAL CLASSES Private Ltd. ‘Bansal Tower’, A-10, Road No.-1, I.P.I.A., Kota-05 Page # 18
PROBABILITY

Illustration :
There are two bags. The first bag contains 5 white and 3 black balls and the second bag contains
3 white and 5 black balls. Two balls are drawn at random from the first bag and are put into the
second bag, without noting their colours. Then two balls are drawn from the second bag. Find the
probability that the balls drawn are white and black.
Sol. 5 White 3 White
3 Black 5 Black
Bag-I Bag-II
Let E1, E2 and E3 be the events of transferring 2 white, 1 white and 1 black, 2 black balls
respectively from the first bag to the second bag.
5
C2 10 5
 P(E1) = 8
 
C2 28 14
5
C1  3C1 5  3 15
P(E2) = 8 = =
C2 28 28

3
C2 3
P(E3) = 8

C2 28
Let A be the event of drawing one white and one black ball from the second bag.
P(A) = P(E1A or E2A or E3A)
= P(E1A) + P(E2A) + P(E3A)

 A  A  A
= P(E1) P   + P(E2) P   + P(E3) P  E 
 E1   E2   3

5 5 C1  5C1 15 4 C1  6 C1 3 3 C1  7 C1
=  10   10   10
14 C2 28 C2 28 C2

5 5 15 8 3 7 673
=      
14 9 28 15 28 15 12600

Illustration :
Two machines A and B produce respectively 60% and 40% of the total numbers of items of a
factory. The percentages of defective output of these machines are respectively 2% and 5%. If an
item is selected at random, what is the probability that the item is (i) defective (ii) non-defective?
Sol. Let E1, E2 be the events of drawing an item produced by machine A and machine B respectively.
Let A be the event of selecting a defective item.
 A represent the event of selecting a non-defective item.
We have
P(E1) = 60% ; P(E2) = 40%
 A
P  E  = Probability that an item produced A is defective = 2%
 1
 A
P  E  = Probability that an item produced by B is defective = 5%
 2

BANSAL CLASSES Private Ltd. ‘Bansal Tower’, A-10, Road No.-1, I.P.I.A., Kota-05 Page # 19
PROBABILITY

(i) P(selected item is defective)


= P(A) = P(E1 A or E2A) = P(E1A) + P(E2A)
 A  A
= P(E1) P  E  + P(E2)  E 
 1  2
= (60%) (2%) + (40%) (5%)
60 2 40 5 320
=    = = 0.032
100 100 100 100 1000
(ii) P(selected item is non-defective)
= P( A ) = P(E1 A or E2 A ) = P(E1 A ) + P(E2 A )
 A  A
= P(E1) P   + P(E2) P  
 E1   E2 
= (60%) (98%) + (40%) (95%)
60 98 40 95 9680
=    = = 0.968
100 100 100 100 10000

BAYE'S THEOREM :
If an event A can occur only with one of the n pairwise mutually exclusive and exhaustive
events B1, B2, .... Bn & if the conditional probabilities of the event.
P(Bi ).PA / Bi 
P(A/B1), P(A/B2) .......P(A/Bn) are known then, P (B1/A) = n

 P(Bi ).PA / Bi 
i1
Proof :
The event Aoccurs with one of the 'n' mutually exclusive e exhaustive events B1, B2, B3, ……, Bn
A = AB1 + AB2 + AB3 + …… + ABn
n
P(A) = P(AB1) + P(AB2) + …… + P(ABn) =  P(AB )
i 1
i

Note :
A = event what we have,
B1 = event what we want,
B1, B2, ……, Bn are alternative events.
B3
Now, B2 Bn–1

 Bi  A B1 A
P(ABi) = P(A) · P   = P(Bi)·P  B 
Bn
A  i

A A A


P(Bi )P  P(Bi ) · P  P(Bi ) · P 
 Bi   Bi  =  Bi   Bi 
P = =
A P (A )
n n
A
 P(ABi )  P(Bi ) · P 
i 1 i 1  Bi 

BANSAL CLASSES Private Ltd. ‘Bansal Tower’, A-10, Road No.-1, I.P.I.A., Kota-05 Page # 20
PROBABILITY

Illustration :
Bag A contains 3 white and 2 black balls. Bag B contains 2 white and 2 black balls. One ball is
drawn at random from A and transferred to B. One ball is selected at random from B and is found
to be white. The probability that the transferred ball is white is
(A) 8/13 (B) 5/13 (C) 4/13 (D) 9/13
Sol. Let E1 and E2 denote the events of the transferred ball being white and black, respectively.
W denotes the drawn ball from B is white. By hypothesis,
3 2
C1 3 C1 2
P( E1 )  5
 , P( E2 )  5

C1 5 C1 5
 W  3C 3  W  2C 2
P   5 1  , P   5 1 
 E1  C1 5  E2  C1 5
By Boyes' theorem

W  3 3
P( E1 )P  
 E1   E1  5 5 9
P  = = =
 W  P( E )P W   P( E )P W  3 3 2 2 13
  
1 E  2 E  5 5 5 5
 1  2
Illustration :
A letter is to come from either LONDON or CLIFTON. The postal mark on the letter legibly
shows consecutive letters "ON". The probability that the letter has come from LONDON is
(A) 12/17 (B) 13/17 (C) 5/17 (D) 4/17
Sol. Let the events be defined as
E1 : Letter coming from LONDON
E2 : Letter coming from CLIFTON
E3 : Two consecutive letters ON.
The word LONDON contains 5 types of consecutive letters (LO, ON, ND, DO, ON) of which there
are two ON's. The word CLIFTON contains 6 types of consecutive letters (CL, LI, IF, FT, TO, ON)
of which there is one "ON". Now

1 E  2 E  1
P(E1) = = P(E2)  P 3   and P 3  
2  E2  5  E2  6
By Boyes' theorem
1 2
 E1  
2 5 12
P   
 E3  1  2  1  1 17
2 5 2 6
Illustration :
In a factory which manufactures bolts, machines A, B and C manufacture respectively 25%, 35%
and 40% of the bolts. Of their outputs, 5, 4 and 2 percent are respectively defective bolt. A bolts
is drawn at random from the product and is found to be defective. What is the probability that it
is manufactured by the machine B?
Sol. Let events B1, B2, B3 be the following
B1 : the bolt is manufactured by machine A
B2 : the bolt is manufactured by machine B

BANSAL CLASSES Private Ltd. ‘Bansal Tower’, A-10, Road No.-1, I.P.I.A., Kota-05 Page # 21
PROBABILITY

B3 : the bolt is manufactured by machine C


Clearly, B1, B2, B3 are mutually exclusive and exhaustive events and hence, they represent a
partition of the sample space.
Let the event E be 'the bolt is defective'.
The event E occurs with B1 or with B2 or with B3 . Given that,
P(B1) = 25% = 0.25, P(B2) = 0.35 and P(B3) = 0.40
Again P(E|B1) = Probability that the bolt drawn is defective given that it is manufactured by
machine
A = 5% = 0.05
Similarly, P(E|B2) = 0.04, P(E | B3) = 0.02
Hence, by Bayes' Theorem, we have
P( B2 ) P( E | B2 )
P(B2 |E) = P( B ) P( E | B )  P( B ) P( E | B )  P( B ) P( E | B )
1 1 2 2 3 3

0.35  0.04 0.0140 28


= = =
0.25  0.05  0.35  0.04  0.40  0.02 0.0345 69

Illustration :
In a test, an examinee either guesses or copies or knows the answer for a multiple choice question
having FOUR choices of which exactly one is correct. The probability that he makes a guess is
1/3 and the probability for copying is 1/6. The probability that his answer is correct, given that he
copied it is 1/8. The probability that he knew the answer, given that his answer is correct is
5 9 24 20
(A) (B) (C) (D)
29 29 29 29
Sol. Let the events be defined as
E1 : Guessing
E2 : Copying
E3 : Knowing
E : Correct answer
By hypothesis,
1 1 1 1 1
P(E1) = , P(E2) = , P(E3) = 1 –  
3 6 3 6 2
E 1
P  E  = (out of four choices only one is correct)
 1 4

 E  1
P  E  =
 2 8

 E 
P  E  = 1
 3

BANSAL CLASSES Private Ltd. ‘Bansal Tower’, A-10, Road No.-1, I.P.I.A., Kota-05 Page # 22
PROBABILITY

Therefore by Bayes' theorem

 E  1
P( E3 )P  1
 E3   E3  2 24
P   = = =
 E  E  E   E  1 1 1 1 1 29
P( E1 )P   P( E2 )P   P( E3 )P  3  4  6  8  2  1
 E1   E2   E3 
Illustration :
Suppose that the reliability of a HIV test is specified as follows :
Of people having HIV, 90% of the test detect the disease but 10% go undetected. Of people free of
HIV, 99% of the test are judged HIV –ive but 1% are diagnosed as showing HIV +ve. From a
large population of which only 0.1% have HIV, one person is selected at random, given the HIV
test, and the pathologist reports him/her as HIV +ve. What is the probability that the person
actually has HIV?
Sol. Let E denote the event that the person selected is actually having HIV and A the event that the
person's HIV test is diagnosed as +ve. We need to find P(E|A). Also E' denotes the event that the
person selected is actually not having HIV.
Clearly, {E, E'} is a partition of the sample space of all people in the population. We are given that
0 .1
P(E) = 0.1% = = 0.001
100
P(E') = 1 – P(E) = 0.999
P(A|E) = P (Person tested as HIV +ve given that he/she is actually having HIV)
90
= 90% = = 0.9
100
and P(A|E') = P(Person tested as HIV +ve given that he/she is actually not having HIV)
1
= 1% = = 0.01
100
Now, by Bayes' theorem
P( E ) P( A | E ) 0.001  0.9 90
P(E|A) = P( E ) P( A | E )  P( E' ) P( A | E' ) = =
0.001  0.9  0.999  0.01 1089
Thus, the probability that a person selected at random is actually having HIV given that he/she is
90
tested HIV +ve is .
1089

PROBABILITIES THROUGH STATISTICAL (STOCHASTIC) TREE


DIAGRAM:
Illustration :

A : box contains three coins A, B and C 1


A : Normal coin; B : Double Headed (DH) coin ; C : a weighted coin so that P(H) =
3
A coin is randomly selected & tossed
(A) Find the probability that head appears.
(B) If head appear find the probability that it is a normal coin P(A/H)
(C) Find the probability that tail appears.
(D) If tail appears, find the probability that it is a weighted coin P(C/T)

BANSAL CLASSES Private Ltd. ‘Bansal Tower’, A-10, Road No.-1, I.P.I.A., Kota-05 Page # 23
PROBABILITY

1 1 1 1 1 11 T
Sol. (A) P(H) = ·  ·1  · = 1/2
3 2 3 3 3 18
A 1/2
1 1 1/3 (N.C)
· H
 A  P( A  B ) 3 2 = 3
(B) P   = = 1/3 1
H P( H ) 11 11 Start B H
(D.H)
18
1/3 2/3 T
C 1/3
1 1 1 1 2 7 (W.C)
(C) P(T) = ·  ·0  · = H
3 2 3 3 3 18
11 7
or 1 – P(H) = 1 – =
18 18
1 2
·
C  P( C  T ) 3 3 4
(D) P  = = 7 =
T  P( T ) 7
18
Illustration :
5 coins DH denoted by say X
A box contains 10 coins 3 coins DT denoted by say Y
2 coins normal denoted by Z
A coin is dream at random from the box and tossed, fall headwise. Find the probability that it was
a normal coin.

1 H
 Z  P( H  Z ) X
Sol. P  = DH 0
H P( H ) T
5/10
0 H
2 1 3/10
5 2 1 6 · 1
Start
DT 1
P(H) = ·1  = = 10 2 = T
10 10  2 10 6 6 N.C.
2/10 1/2 H
10
Z 1/2
T

BANSAL CLASSES Private Ltd. ‘Bansal Tower’, A-10, Road No.-1, I.P.I.A., Kota-05 Page # 24
PROBABILITY

MATHEMATICAL EXPECTATION (PRACTICAL USE OF PROBABILITY


IN DAY TO DAY LIFE):
It is worthwhile indicating that if 'P' represents a person’s chance of success in any venture and 'M 'the
sum of money which he will receive in case of success, then the sum of money denoted by 'P·M' is
called his expectation.

Illustration :
Two players of equal skill A and B are playing a game. They leave off playing (due to some force
majeure conditions) when A wants 3 points and B wants 2 to win. If the prize money is Rs.16000/-.
How can the referee divide the money in a fair way.
Sol. Since, A wins if he scores 3 points before B scores 2.
1
Probability of A's scoring a point = Probability of B's scoring at point =
2
Hence, required probability that A succeeds
1 1 1 1 1 1 1 1 1 1 1 1 1 1 1 5
 · ·  · · ·  · · ·  · · · 
2 2 2 2 2 2 2 2 2 2 2 2 2 2 2 16
5 11
Probability that B succeeds = 1 – =
16 16
5
 A's expectation = × 16000 = 5000
16
11
B's expectation = × 16000 = 11000
16
RANDOM VARIABLE :
Random variables are of two types :
(i) Discrete random variable.
(ii) Continuous random variables.
(i) A random variable is called a discrete random variable if it can take only finitely many values. For
example, in the experiment of drawing three cards from a pack of playing cards, the random variable
"number of kings drawn" is a discrete random variable taking value either 0 or 1 or 2 and 3.
(ii) A random variable is called a continuous random variable if it can take any value between certain limits.
For example, height, weight of students in a class are continuous random variables.

Probability Distribution of a discrete random variable :


Let x be a discrete random variable assuming values x1, x2, x3, ……, xn corresponding to the various
outcomes of a random experiment. If the probability of occurrence of x = xi is P(xi) = pi, 1  i  n such
that p1 + p2 + p3 + …… + pn = 1, then the function, P(xi) = Pi , 1  i  n is called the probability function
of the random variable x and the set {P(x1), P(x2), P(x3),……, P(xn)} is called the probability distribution
of x.

BANSAL CLASSES Private Ltd. ‘Bansal Tower’, A-10, Road No.-1, I.P.I.A., Kota-05 Page # 25
PROBABILITY

Illustration :
Three balls are drawn are by one without replacement from a bag containing 5 white and 4 red
balls. Find the probability distribution of the number of red balls drawn.
Sol. Let x denote the discrete random variable "number of red balls". 5 White
 The possible values of x are 0, 1, 2, 3. 4 Red
Let Ri be the event of drawing a red ball from the bag in the ith draw, i = 1, 2, 3.
R   R  5 4 3 60 5
P(x = 0) = P( R1R2 R3 ) = P ( R1 ) P  2  P  3  =   = =
 R1   R1R2  9 8 7 504 42

P(x = 1) = P R1R2 R3 or R1R2 R3 or R1R2 R3 

 R2   R3  P( R1 )  R2   R3 
= P(R1) P   P   + + P ( R1 ) P  R  P  R R 
 R1   R1R2  R   R   1  1 2
P 2  P 3 
 R1   R1R2 
4 5 4 5 4 4 5 4 4 240 10
=         = =
9 8 7 9 8 7 9 8 7 504 21
P(x = 2) = P R1R2 R3 or R1R2 R3 or R1R2 R3 

 R2   R3  R   R  R   R 
= P(R1) P   P   + P(R ) P  2  P  3  + P ( R1 ) P  2  P  3 
 R1   R1R2 
1
 R1   R1R2   R1   R1R2 
4 3 5 4 5 3 5 4 3 180 5
=         = =
9 8 7 9 8 7 9 8 7 504 14
 R2   R3  4 3 2 24 1
P(x = 3) = P(R1R2R3) = R(R1) P  R  P  R R  =   = =
 1  1 2 9 8 7 504 21

The required Probability distribution is


x 0 1 2 3
5 10 5 1
P( x )
42 21 14 21

BANSAL CLASSES Private Ltd. ‘Bansal Tower’, A-10, Road No.-1, I.P.I.A., Kota-05 Page # 26
PROBABILITY

MEAN AND VARIANCE OF A PROBABILITY DISTRIBUTION :

(1) Mean :-
If a random variable X assumes the values x1, x2, ……, xn with probabilities p1, p2, ……, pn respectively
then the mean of X is defined by
Rendom variable (x i ) Probability (pi ) pi x i
x1 p1 p1x1
x2 p2 p2x 2
  
  
xn pn pn x n
n
 pi x i n  n 
Then, mean () = i 1
n =  x i pi   pi  1 
 i 1 
 pi i 1

i 1

(2) Variance :

=  p x  2x    =  p x 
n n n
2 =  p i ( x i  ) 2
i
2
i i
2 2
i i  2pi x i  pi 2
i 1 i 1 i 1

n n n n
=  pi x i2  2  pi x i   2
 pi =  pi x12  2
i 1 i 1 i 1 i 1

(3) Standard Deviation :


SD = +  2

Illustration :
Two bad eggs are accidently mixed with 10 good eggs 3 eggs are drawn simultaneously from the
basket. Find the mean and variance of the number of bad eggs drawn.
10
C3 10 · 9 · 8 6
Sol. At x = 0, P(0) = 12 = = xi pi pi x i
C3 12 ·11·10 11
6
0 0
2
C1  10C2 9 11
At x = 1, P(1) = 12 = 9 9
C3 22 1
22 22
2
C2 · 10C1 10 ·6 1 1 2
At x = 2, P(2) = 12 = = 2
C3 12 ·11·10 22 22 22
11 1
=  pi xi = =
22 2
9 4
  pi xi2 =0+ +
22 22

BANSAL CLASSES Private Ltd. ‘Bansal Tower’, A-10, Road No.-1, I.P.I.A., Kota-05 Page # 27
PROBABILITY

13 1 15
2 =  pi xi2 – 2 =  =
22 4 44
Ans.

BINOMIAL PROBABILITY DISTRIBUTION :

Let an experiment has n independent trials


and each of the trial has two possible out xi pi pi x i pi x i2
n
comes i.e. success or failure. 0 C0 p 0 q n 0  n C 0 p 0q n 02 · n C0 p 0q n
If getting number of successes in the
1 n
C1p1q n 1 1 n C1p1q n 1 12 · n C1 p1q n 1
experiment is a random variable then
probability of getting exactly r-successes 2 n
C 2 p 2q n  2  
is - 3 n
C 3 p 3 q n 3  
P(x = r) = nCr pr · qn–r
where p = probability of getting success    
and q = probabilityof getting failure r n
Cr p r q n r r  n C r p r ·q n  r r 2 · n Cr p r q n r
Mean of BPD of a random variable
n n n
n
=  pi x i =  r· n r
Cr · p · q n r
=  r · r · n 1Cr1 · pr · q n r = p · n  n 1 Cr 1 · p r 1q n r
r 0 r 0 r 1
= np [n–1C
0 · p0 qn–1 + n–1C 1 · p1qn–2 + …… + n–1C
n–1 pn–1 q0]
= np (p + q)n–1 = np

Variance of BPD of a random variable :

2 =  pi x12  2
n n
n
=  r · Cr · p q  r r · r · n 1Cr 1 · pr · q nr
r n r

2 n
pi x i2 =
n 0 r 0
n n
2 n n 1
=  r · · C r 1 · p · q =  (r  1  1)· n 1Cr1 · pr · q n r
r n r

r 0 r r 0

n n 
  (r  1) · n 1C r 1 · p r · q n  r   n 1 Cr 1 · p r · q n r 
 r 0 r 0 
n n
n · (n – 1)  Cr 2 · p r · q n  r + pn  n 1 Cr 1 · p r · q n  r
n 2
= p2
n r r 1
= p2 · n(n – 1) (p + q)n–2 + pn · (p + q)n–1
= p2 · n(n – 1) + pn
 2 = p2n2 – p2n + pn – n2p2 {  = np}
2 = pn(1 – p) = npq Ans.

BANSAL CLASSES Private Ltd. ‘Bansal Tower’, A-10, Road No.-1, I.P.I.A., Kota-05 Page # 28
PROBABILITY

Standard deviation of BPD of a random variable :


Positive value of square root of variance is called standard deviation.
SD = +  2 = n pq

Illustration :
A pair of dice is thrown 5 times if getting a doublet is considered as a success then find the mean
& variance of the successes.
Sol. Here n = 5 and favourable sample space are (1, 1), (2, 2), (3, 3), (4, 4), (5, 5), (6, 6)
 p = 1/6 and q = 5/6
Mean =  = np = 5/6
Variance = 2 = npq = 25/36
Illustration :
If difference between mean & variance of a BPD is 1 and difference between squares is 11 then
find the probability of getting exactly 3 successes.
Sol. Given
np – npq = 1 ……(i)
n2p2 – n2p2q2 = 11 ……(ii)
 np + npq = 11
npq = 5 q = 5/6 p = 1/6 n = 36
3 33
1 5
 Required probability = 36C    
3 6
  6 

Illustration :

If the sum of the mean and variance of a binomial distribution for 5 trials is 1.8, find the distribution.

Sol. We know that


mean = up and variance = npq
It is being given that n = 5 and mean + variance = 1.8
 np + npq = 1.8, where n = 5
 5p + 5pq = 1.8
 p + p(1 – p) = 0.36 [ q = (1 – p)]
 p – 2p + 0.36 = 0
2

 100 p2 – 200p + 36 = 0
 25p2 – 50p + 9 = 0
 25p2 – 45p – 5p + 9 = 0
 5p (5p – 9) – (5p – 9) = 0
 (5p – 9) (5p – 1) = 0
1
 p= = 0.2 [ p cannot exceed 1]
5
Thus, n = 5, p = 0.2, and q = (1 – p) = (1 – 0.2) = 0.8
Let X denote the binomial variate. Then, the required distribution is
P(X = r) = nCr · pr · q(n–r) = 5Cr · (0.2)r · (0.8)(5–r) where r = 0, 1, 2, 3, 4, 5

BANSAL CLASSES Private Ltd. ‘Bansal Tower’, A-10, Road No.-1, I.P.I.A., Kota-05 Page # 29
PROBABILITY

EXERCISE-1 (SPECIAL DPP)

SPECIAL DPP-1

Q.1 6 married couples are standing in a room. If 4 people are chosen at random, then the chance that exactly
one married couple is among the 4 is :
16 8 17 24
(A) (B) (C) (D)
33 33 33 33

Q.2 A quadratic equation is chosen from the set of all the quadratic equations which are unchanged by
squaring their roots. The chance that the chosen equation has equal roots is :
(A) 1/2 (B) 1/3 (C) 1/4 (D) 2/3

Q.3 The probability that a two digit positive number selected at random has its tens digit at least three more
than its unit digit is
(A) 14/45 (B) 7/45 (C) 36/45 (D) 1/6

Q.4 There are 20 businessmen sitting around a circular table holding a meeting. Three of the men are chosen
to bring snacks. Let P denotes the probability that no two of the three men that were chosen, sat next to
each other then P lies in the interval
1 1 1 1 1 1 2 3
(A)  ,  (B)  ,  (C)  ,  (D)  , 
5 4  4 3 6 5 3 4

Q.5 4 integers are randomly selected from the numbers from 1 to 10. The chance that there are atleast two
successive numbers among those 4 selected, is
5 3 2 1
(A) (B) (C) (D)
6 4 3 2

Q.6 There are 5 balls of different colours and for each ball there is a box of the same colour as that of the ball.
a
If the balls are distributed randomly one in each box and P = is the probability that not more than one
b
ball go into the box of its own colour, the least value of (a + b) is
(A) 119 (B) 209 (C) 229 (D) none

Q.7 Mr. Aforgot to write down a very important phone number.All he remembers is that it started with 713
and that the next set of 4 digit involved are 1, 7 and 9 with one of these numbers appearing twice.
He guesses a phone number and dials randomly. The odds in favour of dialing the correct telephone
number, is
(A) 1 : 35 (B) 1 : 71 (C) 1 : 23 (D) 1 : 36

Q.8 There are 8 students from 4 schools A, B, C, D, 2 students from each school. Let these 8 students enter
in 4 rooms R1, R2, R3, R4, so that each room will have 2 students. The probability that each room have
students from the same school, is
1 2 3 4
(A) (B) (C) (D)
105 105 105 105
BANSAL CLASSES Private Ltd. ‘Bansal Tower’, A-10, Road No.-1, I.P.I.A., Kota-05 Page # 30
PROBABILITY

Q.9 A fair die is tossed. If the number is odd, then the probability that it is prime is p1 and again three fair
coins are tossed. If both heads and tails appear, then the probability that exactly one head appears is p2,
then which of the following is correct?
1
(A) p1 < p2 (B) p1 = p2 (C) p1p2 = (D) p1 + p2 < 1
3
Q.10 Mr. Alives at origin on the cartesian plane and has his office at (4, 5). His friend lives at (2, 3) on the
same plane. Mr.A can go to his office travelling one block at a time either in the + y or + x direction. If
all possible paths are equally likely then the probability that Mr.Apassed his friends house is
(A) 1/2 (B) 10/21 (C) 1/4 (D) 11/21

Q.11 I have 3 normal dice, one red, one blue and one green and I roll all three simultaneously. Let P be the
probability that the sum of the numbers on the red and blue dice is equal to the number on the green die.
If P is the written in lowest terms as a/b then the value of (a + b) equals
(A) 79 (B) 77 (C) 61 (D) 57

Q.12 A cube with all six faces coloured is cut into 64 cubical blocks of the same size which are thoroughly
mixed. Find the probability that the 2 randomly chosen blocks have 2 coloured faces each.

Q.13 Thirteen persons take their places at a round table, Find the odds against two particular persons sitting
together.

Q.14 Let S denotes the set of nine digit numbers whose digits are 1, 2, 3, 4, 5, 6, 7, 8, 9 such that each of these
digit occurs exactly once. A nine digit number is chosen randomly. Five events defined on S are E-1,
E-2, E-3, E-4 and E-5 which are described in Column-I.
Match the probabilities of these events which are given in Column-II.
Column-I Column-II
4
(A) E-1: The number chosen is even (P)
63
7
(B) E-2: The first, fifth and ninth digits of the chosen number will be odd (Q)
9
4
(C) E-3: The number will be greater than 3 · 108 (R)
9
(D) E-4: The sum of digits used in the number on first 5
two places from the left, equals the digit used in the last place (S)
42
(E) E-5: The number chosen is divisible by 6

SPECIAL DPP-2

Q.1 Given two independent events A, B such that P (A) = 0.3, P (B) = 0.6. Determine
(i) P (A and B) (ii) P (A and not B) (iii) P (notAand B)
(iv) P (neitherAnor B) (v) P (A or B)

Q.2 A coin is tossed and a die is thrown. Find the probability that the outcome will be a head or a number
greater than 4.

Q.3 A coin is biased so that heads is three times as likely to appear as tails. Find P(H) and P(T). If such a coin
is tossed twice find the probability that head occurs at least once.
BANSAL CLASSES Private Ltd. ‘Bansal Tower’, A-10, Road No.-1, I.P.I.A., Kota-05 Page # 31
PROBABILITY

Q.4 3 studentsAand B and C are in a swimming race.Aand B have the same probability of winning and each
is twice as likely to win as C. Find the probability that B or C wins. Assume no two reach the winning
point simultaneously.

Q.5 Consider the following three hands of five cards, dealt from an ordinary deck of 52 cards
H1: All four queens and a two.
H2: The ace, king, queen, jack and ten, all in the same suit.
H3: The ten of hearts, the ten of clubs, and three kings.
(A) H1 is more likely than H2 or H3. (B) H2 is more likely than H1 or H3.
(C) H3 is more likely than H1 or H2. (D)All three hands (H1, H2 & H3) are equally likely.

Q.6 SouthAfrican cricket captain lost the toss of a coin 13 times out of 14. The chance of this happening was
7 1 13 13
(A) (B) (C) (D)
213 213 214 213

Q.7 If two of the 64 squares are chosen at random on a chess board, the probability that they have a side in
common is :
(A) 1/9 (B) 1/18 (C) 2/7 (D) none

Q.8 Of all the functions that can be defined from the set A : {1, 2, 3, 4}  B(5, 6, 7, 8, 9}, a function is
randomly selected. The chance that the selected mapping is strictly monotonic, is
1 2 5 5
(A) (B) (C) (D)
125 125 4096 2048

Q.9 Let A & B be two events. Suppose P(A) = 0.4 , P(B) = p & P(A  B) = 0.7. The value of p for which
A & B are independent is :
(A) 1/3 (B) 1/4 (C) 1/2 (D) 1/5

Q.10 Let a red die, a blue die, a green die and a white die are rolled once, the dice being fair. The outcomes
on the red, blue, green and white die denote the numbers a, b, c and d respectively. Let E denotes the
event that absolute value of E = (a – 1)(b – 2)(c – 3)(d – 6) = 1, then P(E) is
1 1 2 1
(A) (B) (C) (D)
324 648 324 162

Q.11 A jar contains 2 yellow candies, 4 red candies, and 6 blue candies. Candies are randomly drawn out of
the jar one-by-one and eaten. The probability that the 2 yellow candies will be eaten before any of the
red candies are eaten, is
1 1 1 1
(A) (B) (C) (D)
18 15 24 36

Q.12 A party consisting of 5 boys and 10 girls is divided at random into five groups of three persons each.
The chance that each group will have one boy is 81 n  where n  N is divisible by
(A) 5 (B) 7 (C) 11 (D) 13

BANSAL CLASSES Private Ltd. ‘Bansal Tower’, A-10, Road No.-1, I.P.I.A., Kota-05 Page # 32
PROBABILITY

Q.13 An urn contains 4 blue, 6 red and 8 green balls. A random sample of 4 balls is drawn. Which of the
following hold(s) good?
3
(A) The chance that the sample contains two blue balls is .
17
11
(B) The chance that the sample contains two red balls is .
34
1
(C) The chance that the sample contains two blue balls if it contains two red balls is .
11
(D) The chance that the sample contains atleast one ball of each colour is more than 50%.

Q.14 5 different marbles are placed in 5 different boxes randomly. Find the probability that exactly two boxes
remain empty. Given each box can hold any number of marbles.

SPECIAL DPP-3

Q.1 Astudents obtained the probabilities of the two events Aand B as follows
2 3 23
P(Ac) = ; P(B) = ; P(Ac  Bc) = then
3 4 24
1 17
(A) P(A  B) = (B) P (Ac  B) =
24 24
7 the students has surely made some
(C) P (A  Bc) = (D) mistake in obtaining the said probabilities
24

Q.2 If A & B are two independent events, each with probability p , (p  0) then P A A  B is:
(A) 1/p (B) 1/2 (C) 2/p (D) 1/(2  p)

1 2
Q.3 If A and B are two independent events such that P(A) = and P(B) = , then
2 3
P  (A  B) (A  B) ( A  B)  has the value equal to
1 1 1 2
(A) (B) (C) (D)
3 4 2 3

Q.4 A pair of numbers is picked up randomly (without replacement) from the set
{1, 2, 3, 5, 7, 11, 12, 13, 17, 19}. The probability that the number 11 was picked given that the sum of
the numbers was even, is nearly :
(A) 0.1 (B) 0.125 (C) 0.24 (D) 0.18

Q.5 Each coefficient of the quadratic equation Ax2 + Bx + C = 0 is determined by the score thrown by an
a
unbiased six faced die having numbers 1 to 6. If P = (a, b  N) denotes the probability that the
b
equation has equal roots, the least value of (a + b) is
(A) 37 (B) 55 (C) 73 (D) 221
BANSAL CLASSES Private Ltd. ‘Bansal Tower’, A-10, Road No.-1, I.P.I.A., Kota-05 Page # 33
PROBABILITY

Q.6 A card is drawn & replaced in an ordinary pack of 52 playing cards. Minimum number of times must a
card be drawn so that there is atleast an even chance of drawing a heart, is
(A) 2 (B) 3 (C) 4 (D) more than four

Q.7 Whenever horses a, b, c race together, their respective probabilities of winning the race are 0.3, 0.5 and
0.2 respectively. If they race three times the probability that “the same horse wins all the three races” and
the probability that a, b, c each wins one race, are respectively (Assume no dead heat)
8 9 16 3 12 15 10 8
(A) ; (B) , (C) ; (D) ;
50 50 100 100 50 50 50 50

Q.8 Two cubes have their faces painted either red or blue. The first cube has five red faces and one blue face.
When the two cubes are rolled simultaneously, the probability that the two top faces show the same
colour is 1/2. Number of red faces on the second cube, is
(A) 1 (B) 2 (C) 3 (D) 4

Q.9 An urn contains 3 red balls and n white balls.


Mr. Adraws two balls together from the urn. The probability that they have the same colour is 1 2 .
Mr. B draws one ball from the urn, notes its colour and replaces it. He then draws a second ball from the
urn and finds that both balls have the same colour is, 5 8 . The possible value of n is
(A) 9 (B) 6 (C) 5 (D) 1

Q.10 In a table tennis singles, two players play, and one of them must win. Probability thatAbeats B is p, B
beats C is q and C beatsAis r. If B plays with C and then the winner plays withA, then the probability
thatAwill be the final winner, is
(A) qp + (1 – q)r (B) (1 – r)q + pq
(C) pq + (1 – q)(1 – r) (D) qp + (1 – r)

Q.11 If E & F are events with P(E)  P(F) & P(E  F) > 0, then
(A) occurrence of E  occurrence of F
(B) occurrence of F  occurrence of E
(C) non  occurrence of E  non  occurrence of F
(D) none of the above implications holds.

Q.12 Events A and C are independent. If the probabilities relatingA, B and C are P (A) = 1/5; P (B) = 1/6;
P(A  C) = 1/20; P(B  C) = 3/8 then
(A) events B and C are independent
(B) events B and C are mutually exclusive
(C) events B and C are neither independent nor mutually exclusive
(D) events Aand C are equiprobable

7 4 1
Q.13 Events A and B satisfy P( A) = , P (B) = and P A  B = .
10 10 10
Which of the following statement(s) is/are CORRECT?

(A) 15 P(A  B)  7 PB A  (B) PA  B A  B 


1
2
2
(C) PA B  PB A  (D) P (neither A nor B occurs) =
5

BANSAL CLASSES Private Ltd. ‘Bansal Tower’, A-10, Road No.-1, I.P.I.A., Kota-05 Page # 34
PROBABILITY

a b 
Q.14 Let M = c d  where a, b, c, d  {1, 2}, then
 
1
(A) probability that M is a symmetric matrix is .
2
5
(B) probability that M is a non-singular matrix is .
16
3
(C) probability that M is a non-singular matrix is .
16
1
(D) probability that trace of M is an even number is .
2
[Note : The trace of a matrix is the sum of its diagonal entries.]

Q.15 A supermarket offers ice creams in 6 different varieties. Ice creams of each variety are unlimited and
identical. Which of the following is/are correct?
(A) Number of ways in which four ice-creams can be selected is 126.
(B) Number of ways in which four ice-creams can be selected if atleast two ice-creams are of same
variety is 111.
(C) If four ice-creams are randomly selected then the probability that all four ice-creams are of different
5
variety is .
42
(D) If four ice-creams are randomly selected then the probability that two ice-creams are of same
5
variety and other two ice-creams are also of another same variety is .
42

Q.16 In a game of chance each player throws two unbiased dice and scores the difference between the larger
and smaller number which arise. Two players compete and one or the other wins if and only if he scores
p
atleast 4 more than his opponent. If the probability that neither player wins is given by , then find the
q
least value of (p + q) where p, q  N.

2
Q.17 The probability that a person will get an electric contract is and the probability that he will not get
5
4 2
plumbing contract is . If the probability of getting at least one contract is , what is the probability that
7 3
he will get both?

Q.18 The odds that a book will be favourably reviewed by three independent critics are 5 to 2, 4 to 3, and 3
to 4 respectively. What is the probability that of the three reviews a majority will be favourable?

BANSAL CLASSES Private Ltd. ‘Bansal Tower’, A-10, Road No.-1, I.P.I.A., Kota-05 Page # 35
PROBABILITY

SPECIAL DPP-4

Q.1 One bag contains 3 white & 2 black balls, and another contains 2 white & 3 black balls.Aball is drawn
from the second bag & placed in the first, then a ball is drawn from the first bag & placed in the second.
When the pair of the operations is repeated, the probabilitythat the first bag will contain 5 white balls is:
(A) 1/25 (B) 1/125 (C) 1/225 (D) 2/15

Q.2 A child throws 2 fair dice. If the numbers showing are unequal, he adds them together to get his final
score. On the other hand, if the numbers showing are equal, he throws 2 more dice & adds all 4 numbers
showing to get his final score. The probability that his final score is 6 is:
145 146 147 148
(A) (B) (C) (D)
1296 1296 1296 1296
Q.3 A bag contains 3 R & 3 G balls and a person draws out 3 at random. He then drops 3 blue balls into the
bag & again draws out 3 at random. The chance that the 3 later balls being all of different colours is
(A) 15% (B) 20% (C) 27% (D) 40%

Q.4 A biased coin with probability P, 0 < P< 1, of heads, is tossed until a head appears for the first time. If
the probability that the number of tosses required is even is 2/5, then the value of P is
(A ) 1/4 (B) 1/6 (C) 1/3 (D) 1/2

Q.5 Two numbers a and b are selected from the set of natural number then the probability that (a2 + b2) is
divisible by5 is
9 7 11 1
(A) (B) (C) (D)
25 18 36 3

Q.6 When a missile is fired from a ship, the probability that it is intercepted is 1/3. The probability that the
missile hits the target, given that it is not intercepted is 3/4. If three missiles are fired independently from
the ship, the probability that all three hit the target, is
(A) 1/12 (B) 1/8 (C) 3/8 (D) 3/4
Q.7 An urn contains 10 balls coloured either black or red. When selecting two balls from the urn at random,
the probability that a ball of each colour is selected is 8 15 .Assuming that the urn contains more black
balls than red balls, the probability that at least one black ball is selected, when selecting two balls, is
18 30 39 41
(A) (B) (C) (D)
45 45 45 45
Q.8 Afair die is tossed repeatedly. Mr.Awins if it is 1 or 2 on two consecutive tosses and Mr. B wins if it is
3, 4, 5 or 6 on two consecutive tosses. The probability that Awins if the die is tossed indefinitely, is
1 5 1 2
(A) (B) (C) (D)
3 21 4 5
Q.9 An unbiased die with the numbers 1, 2, 3, 4, 6 and 8 on its six faces is rolled. After this roll if an odd
number appears on the top face, all odd numbers on the die are doubled. If an even number appears on
the top face, all the even numbers are halved. If the given die changes in this way then the probability that
the face 2 will appear on the second roll is
(A) 2/18 (B) 3/18 (C) 2/9 (D) 5/18

BANSAL CLASSES Private Ltd. ‘Bansal Tower’, A-10, Road No.-1, I.P.I.A., Kota-05 Page # 36
PROBABILITY

Q.10 Two loaded dice each have the property that the face 2 or 4 is three times as likely to appear as
the face 1, 3, 5 or 6 on each roll. When two such dice are rolled, the probabilityof obtaining a total of 7, is
1 1 7 7
(A) (B) (C) (D)
8 7 50 25

Q.11 A butterfly randomly lands on one of the six squares of the T-shaped
figure shown and then randomly moves to an adjacent square. The
probability that the butterfly ends up on the R square is
(A) 1/4 (B) 1/3 (C) 1/5 (D) 2/5

Q.12 A fair coin is tossed a large number of times. Assuming the tosses are independent which one of the
following statement, is True?
(A) Once the number of flips is large enough, the number of heads will always be exactly half of the total
number of tosses. For example, after 10,000 tosses one should have exactly 5,000 heads.
(B) The proportion of heads will be about 1/2 and this proportion will tend to get closer to 1/2 as the
number of tosses increases.
(C)As the number of tosses increases, any long run of heads will be balanced by a corresponding run of
tails so that the overall proportion of heads is exactly 1/2
(D)All of the above
1 1 1
Q.13 Let A and B be two events such that P (A  B) = , P(A  B) = and P( A) = , where A stands
6 4 4
for complement of event A. Then, events Aand B are
(A) mutuallyexclusive and independent. (B) independent but not equally likely.
(C) equally likelybut not independent. (D) equallylikelyand mutuallyexclusive.

Q.14 In one day test match between India andAustralia the umpire continues tossing a fair coin until the two
consecutive throws either H T or TT are obtained for the first time. If it is H T, India wins and if it is T T,
Australia wins.
Statement-1: Both India andAustralia have equal probability of winning the toss.
Statement-2: If a coin is tossed twice then the events HT or TT are equiprobable.
(A) Statement-1 is true, statement-2 is true and statement-2 is correct explanation for statement-1.
(B) Statement-1 is true, statement-2 is true and statement-2 is NOT the correct explanation for statement-1.
(C) Statement-1 is true, statement-2 is false.
(D) Statement-1 is false, statement-2 is true.
Q.15 Which of the following statement(s) is/are correct?
(A) 3 coins are tossed once. Two of them atleast must land the same way. No mater whether they land
heads or tails, the third coin is equally likely to land either the same way or oppositely. So, the chance
that all the three coins land the same way is 1/2.
(B) Suppose an urn contains 'w' white and 'b' black balls and a ball is drawn from it and is replaced along
with 'd' additional balls of the same colour. Now a second ball is drawn from it. The probability that
the second drawn ball is white is independent of the value of 'd'.
1
(C) A and B are two independent events. If the probability that both A and B occur is
12
1 1
and the probability that neither A nor B occurs is , then P(A – B) = .
2 6
(Given: P(A) < P(B) )
(D) For two given events A and B, P(A) + P(B) – 1  P (A  B)  P(A) + P(B)
BANSAL CLASSES Private Ltd. ‘Bansal Tower’, A-10, Road No.-1, I.P.I.A., Kota-05 Page # 37
PROBABILITY

Q.16 Which of the following statement(s) always hold(s) good?


   
(A) P A B  P A  B denotes the probability that exactly one of the two events Aor B occurs.

(B) For any two events A and B, P ( A )  P AB   P A  B = 1. 
(C) Two mutually exclusive events Aand B for which P(A) > 0 and P(B) > 0 can be independent.
(D) If P B A  = P(B/A) then A and B are independent.

Q.17 A is one of the 6 horses entered for a race, and is to be ridden by one of two jockeys B or C. It is 2 to
1 that B ridesA, in which case all the horses are equally likely to win; if C ridesA, his chance is trebled,
what are the odds against his winning?

Q.18 A child flips 7 fair coins. There are relatively prime positive integers m and n so that  m n  is the
probability that atleast two heads occur, given that atleast three tails occur. Find (m + n).

SPECIAL DPP-5

Q.1 Indicate the correct order sequence in respect of the following :


I. If the probabilitythat a computer will fail during the first hour of operation is 0.01, then if we turn
on 100 computers, exactly one will fail in the first hour of operation.
II. A, B, C simultaneously satisfy P(ABC) = P(A)·P(B)·P(C) and P( ABC ) = P(A)·P(B)· P( C )
and P( AB C) = P(A)· P( B) ·P(C) and P( ABC) = P( A) ·P(B)·P(C) then A, B, C are
independent.
III. Given the events A and B in a sample space. If P(A) = 1, then A and B are independent.
IV. When a fair six sided die is tossed on a table top, the bottom face can not be seen. The probability
that the product of the numbers on the five faces that can be seen is divisible by 6 is one.
(A) FTFT (B) FTTT (C) TFTF (D) TFFF

Q.2 If a, b and c are three numbers (not necessarily different) chosen randomly and with replacement from
the set {1, 2, 3, 4, 5}, the probability that (ab + c) is even, is
35 59 64 75
(A) (B) (C) (D)
125 125 125 125

Q.3 Let A, B, C be three events in a probability space. Suppose that P(A) = 0.5, P(B) = 0.3, P(C) = 0.2,
P(A  B) = 0.15, P(A  C) = 0.1 and P(B  C) = 0.06.
The smallest possible value of P(Ac  Bc  Cc) is
[Note: Ac denotes compliment of event A]
(A) 0.31 (B) 0.25 (C) 0 (D) 0.26

Q.4 Lot A consists of 3G and 2D articles. Lot B consists of 4G and 1D article. A new lot C is formed by
taking 3 articles from A and 2 from B. The probability that an article chosen at random from C is
defective, is
1 2 8
(A) (B) (C) (D) none
3 5 25
BANSAL CLASSES Private Ltd. ‘Bansal Tower’, A-10, Road No.-1, I.P.I.A., Kota-05 Page # 38
PROBABILITY

Q.5 Mr. A and Mr. B each have a bag that contains one ball of each of the colours blue, green, orange, red
and violet. 'A' randomly selects one ball from his bag and puts it into B's bag. 'B' then randomly selects
one ball from his bag and puts it intoA's bag. The probability that after this process the contents of the
two bags are the same, is
(A) 1/6 (B) 1/5 (C) 1/3 (D) 1/2

Q.6 Abox contains 10 tickets numbered from 1 to 10. Two tickets are drawn one byone without replacement.
The probability that the "difference between the first drawn ticket number and the second is not less than
4" is
7 14 11 10
(A) (B) (C) (D)
30 30 30 30

Q.7 A coin has probability 'p' of showing head when tossed. It is tossed n times. Let Pn denotes the probability
that no two (or more) consecutive heads occur. The value of P4 is given by
(A) 1 – 4p2 + 4p3 (B) 1 – 3p2 + 2p3 (C) 1 + 2p2 – 8p3 (D) 1 + p2 – 6p3

Q.8 Two busesAand B are scheduled to arrive at a town central bus station at noon. The probability that bus
A will be late is 1/5. The probability that bus B will be late is 7/25. The probability that the bus B is late
given that busAis late is 9/10. Then the probabilities
(i) neither bus will be late on a particular day and
(ii) bus Ais late given that bus B is late, are respectively
(A) 2/25 and 12/28 (B) 18/25 and 22/28 (C) 7/10 and 18/28 (D) 12/25 and 2/28

Q.9 Shalu bought two cages of birds : Cage-I contains 5 parrots and 1
owl, and Cage-II contains 6 parrots, as shown
One day Shalu forgot to lock both cages and two birds flew from
Cage-I to Cage-II. Then two birds flew back from Cage-II to
Cage-I.Assume that all birds have equal chance of flying, the
probability that the Owl is still in Cage-I, is
(A) 1/6 (B) 1/3 (C) 2/3 (D) 3/4

Paragraph for question for 10 & 11


Two basket ball players each make two attempts of throwing a ball into the basket. The probabilities of
2 3
making a basket at first and second throw are and respectively for both the players. Let
3 4
E1 be the event that both will have the same number of baskets.
E2 be the event that first player will have more baskets than the second.
Q.10 The chance that the event E1 occurs, is
1 31 17 13
(A) (B) (C) (D)
2 72 36 36

Q.11 The chance that the event E2 occurs, is


5 41 47 53
(A) (B) (C) (D)
144 144 144 144

BANSAL CLASSES Private Ltd. ‘Bansal Tower’, A-10, Road No.-1, I.P.I.A., Kota-05 Page # 39
PROBABILITY

Paragraph for question nos. 12 & 13

Suppose you have 10 keys and you wish to open a door and try the keys one at a time, randomly until
you open the door. Only one of the keys will open the door.
Q.12 If you eliminate each unsuccessful trials then the probability that the door gets opened on the 5th trial, is
given by
4
1 1 1 10 9 1
(A) (B) (C) (D) C5 ·   ·  
10 5 2  10   10 

Q.13 If you do not eliminate the each unsuccessful trial from the remaining keys then the probability that the
door gets opened on the 5th trial, is given by
4 4
1 1 10 9 1 9 1
(A) (B) (C) C5 ·   · (D)   ·
2 10  10  10  10  10

Paragraph for question nos. 14 to 16


A JEE aspirant estimates that she will be successful with an 80 percent chance if she studies 10 hours per
day, with a 60 percent chance if she studies 7 hours per day and with a 40 percent chance if she studies
4 hours per day. She further believes that she will study 10 hours, 7 hours and 4 hours per day with
probabilities 0.1, 0.2 and 0.7, respectively
Q.14 The chance she will be successful, is
(A) 0.28 (B) 0.38 (C) 0.48 (D) 0.58

Q.15 Given that she is successful, the chance she studied for 4 hours, is
6 7 8 9
(A) (B) (C) (D)
12 12 12 12

Q.16 Given that she does not achieve success, the chance she studied for 4 hour, is
18 19 20 21
(A) (B) (C) (D)
26 26 26 26

Paragraph for Question no. 17 & 18

32 coins have been distributed randomly into 8 purses, 4 coins into each purse.

Q.17 The chance that two specified coins are there in the same purse, is
1 3 5 7
(A) (B) (C) (D)
32 31 31 32

Q.18 Given that first three purses examined do not contain either of the specified coins, the chance that these
specified coins are there in the same purse, is
5 4 3
(A) (B) (C) (D) none
21 9 19

BANSAL CLASSES Private Ltd. ‘Bansal Tower’, A-10, Road No.-1, I.P.I.A., Kota-05 Page # 40
PROBABILITY

Q.19 Anormal coin is flipped four times. Two events E and F are defined as
E : "more heads than tails occurred"
F : "First two results are different "
which of the following is/are true?
5 8 4 2
(A) P(E) = (B) P(F) = (C) P(E/F) = (D) P(F/E) =
16 16 16 16

Q.20 Consider four defective items : one item has the paint damaged, the second item has a dent, the third has
a scratch and the fourth has all the three defects as mentioned.
Let E be the event that an item randomly selected has its paint damaged and
F be the event that an item randomly selected has a dent and
G be the event that an item randomly selected has a scratch.
then which of the following is/are correct ?
(A ) P (E  F  G) = P(E  F) (B) P(E) = P(F) = P(G) = P(F  G)
(C) E, F and G are independent in pairs (D) P(E/FG) is a sure event.

Q.21 A manufacturer of airplane parts makes a certain engine that has a probability p of failing on any given
flight. There are two planes fitted with this type of engine. One plane has 3 such engines and other plane
has 5.Aplane crashes if more than half the engines fitted in it fail. If the two plane models have the same
probability of crashing then the value of p can be
(A) 0 (B) 1 (C) 1/2 (D) 3/5

SPECIAL DPP-6

Q.1 A letter has either come from LONDON or CLIFTON, but on the post mark 2 consecutive letters
'ON' are found to be visible. The probability that the letter has come from LONDON is
4 2 12 7
(A) (B) (C) (D)
9 3 17 10

Q.2 In a certain factory, machines A, B and C produce bolts. Of their production, machines A, B, and C
produce 2%, 1% and 3% defective bolts respectively. MachineAproduces 35% of the total output of
bolts, machine B produces 25% and machine C produces 40%. Abolts is chosen at random from the
factory's production and is found to be defective. The probability it was produced on machine C, is
6 23 24 3
(A) (B) (C) (D)
11 45 43 11

Q.3 Mr. Dupont is a professional wine taster. When given a French wine, he will identify it with probability
0.9 correctly as French, and will mistake it for a Californian wine with probability 0.1. When given a
Californian wine, he will identify it with probability 0.8 correctly as Californian, and will mistake it for a
French wine with probability 0.2. Suppose that Mr. Dupont is given ten unlabelled glasses of wine, three
with French and seven with Californian wines. He randomly picks a glass, tries the wine, and solemnly
says : "French". The probability that the wine he tasted was Californian, is nearly equal to
(A) 0.14 (B) 0.24 (C) 0.34 (D) 0.44

BANSAL CLASSES Private Ltd. ‘Bansal Tower’, A-10, Road No.-1, I.P.I.A., Kota-05 Page # 41
PROBABILITY

Q.4 A card has been misplaced from the pack of 52 playing cards. From such an incomplete pack, two
cards are drawn simultaneously without replacement and found to be both spades. The probability that
the missing card was also from spade suit, is
13 4 11 11
(A) (B) (C) (D)
48 17 24 50

Q.5 A purse contains 2 six sided dice. One is a normal fair die, while the other has 2 ones, 2 threes, and 2
fives.Adie is picked up and rolled. Because of some secret magnetic attraction of the unfair die, there is
75% chance of picking the unfair die and a 25% chance of picking a fair die. The die is rolled and shows
up the face 3. The probability that a fair die was picked up, is
1 1 1 1
(A) (B) (C) (D)
7 4 6 24

Q.6 On a Saturdaynight 20% of all drivers in U.S.A. are under the influence of alcohol. The probability that
a driver under the influence of alcohol will have an accident is 0.001. The probability that a sober driver
will have an accident is 0.0001. If a car on a saturday night smashed into a tree, the probability that the
driver was under the influence of alcohol, is
(A) 3/7 (B) 4/7 (C) 5/7 (D) 6/7

Q.7 There are three main political parties namely 1, 2, 3. If in the


adjoining table pij , (i, j=1, 2, 3) denote the probability that
party j wins the general elections contested when party i is in
the power. The probability that the party 2 will be in power
after the next two elections, given that the party 1 is in
the power, is
(A) 0.27 (B) 0.24 (C) 0.14 (D) 0.06

Paragraph for question nos. 8 to 10

A box contains b red balls, '2b' white balls and '3b' blue balls where b is a positive integer. 3 balls are
selected at random from the box.

Q.8 If balls are drawn without replacement and 'A' denotes the event that "No two of the selected balls have
the same colour" then
(A) there is no value of b for which P(A) = 0.3
(B) There is exactly one value of b for which P(A) = 0.3 and this value is less than 5.
(C) There is exactly one value of b for which P(A) = 0.3 and this value is greater than 5.
(D) There are two values of b for which P(A) = 0.3

Q.9 If balls are drawn without replacement and 'B' denotes the event that "No two of the 3 drawn balls are
blue" then
1 2
(A) P(B) = if b = 1 (B) P(B) = if b = 2
3 3
1 1
(C) P(B) = if b = 4 (D) P(B) = for all value of b.
4 2

BANSAL CLASSES Private Ltd. ‘Bansal Tower’, A-10, Road No.-1, I.P.I.A., Kota-05 Page # 42
PROBABILITY

Q.10 If P(A) = 0.3, then the value of P(A/B) equals


(A) 3/5 (B) 3/10 (C) 1/2 (D) 2/3

Q.11 The probability that a student passes in Mathematics, Physics and Chemistry are m, p and c respectively.
Of these subjects, the student has a 75% chance of passing in atleast one, a 50% chance of passing in
atleast two, and a 40% chance of passing in exactly two, which of the following relations are true?
19 27 1 1
(A) p + m + c = (B) p + m + c = (C) pmc = (D) pmc =
20 20 10 4

Q.12 Two whole numbers are randomly selected and multiplied. Consider two events E1 and E2 defined as
E1 : Their product is divisible by 5
E2: Unit's place in their product is 5.
Which of the following statement(s) is/are correct?
(A) E1 is twice as likely to occur as E2. (B) E1 and E2 are equiprobable.
(C) P(E2/E1) = 1/4 (D) P(E1/E2) = 1

Q.13 There are three coins C1, C2 and C3, C1 is a fair coin painted blue on the head side and white on the tail
side. C2 and C3 are biased coins so that the probability of a head is p. They are painted blue on the tail
side and red on the head side. Two of the three coins are selected at random and tossed. If the probability
29
that both the coins land up with sides of the same colour is then the possible value(s) of p can be
96
1 3 5 7
(A) (B) (C) (D)
8 8 8 8

Q.14 A, B are two inaccurate arithmeticians whose chance of solving a given question correctly are (1/8) and
(1/12) respectively. They solve a problem and obtained the same result. If it is 1000 to 1 against their
a
making the same mistake. If the chance that the result is correct is expressed as , find the least value
b
of (a + b).

SPECIAL DPP-7

1 1 1
Q.1 Suppose families always have one, two or three children, with probabilities , and respectively..
4 2 4
Assume everyone eventually gets married and has children, the probability of a couple having exactly
four grandchildren is
27 37 25 20
(A) (B) (C) (D)
128 128 128 128

Q.2 Miss C has either Tea or Coffee at morning break. If she has tea one morning, the probability she has tea
the next morning is 0.4. If she has coffee one morning, the probability she has coffee next morning is 0.3.
Suppose she has coffee on a Mondaymorning.The probabilitythat she has tea on the followingWednesday
morningis
(A) 0.46 (B) 0.49 (C) 0.51 (D) 0.61

BANSAL CLASSES Private Ltd. ‘Bansal Tower’, A-10, Road No.-1, I.P.I.A., Kota-05 Page # 43
PROBABILITY

Q.3 In a Maths paper there are 3 sections A, B & C. Section A is compulsory. Out of sections B & C a
student has to attempt any one. Passing in the paper means passing in A & passing in B or C. The
probability of the student passing in A, B & C are p, q & 1/2 respectively. If the probability that the
student is successful is 1/2 then :
(A) p = q = 1 (B) p = q = 1/2 (C) p = 1, q = 0 (D) p = 1, q = 1/2

Q.4 The number 'a' is randomly selected from the set {0, 1, 2, 3, ...... 98, 99}. The number 'b' is selected
from the same set. Probability that the number 3a + 7b has a digit equal to 8 at the units place, is
1 2 4 3
(A) (B) (C) (D)
16 16 16 16

Q.5 On a normal standard die one of the 21 dots from any one of the six faces is removed at random with
each dot equally likely to be chosen. The die is then rolled. The probability that the top face has an odd
number of dots is
5 5 11 6
(A) (B) (C) (D)
11 12 21 11

Q.6 Two boys A and B find the jumble of n ropes lying on the floor. Each takes hold of one loose end
1
randomly. If the probability that they are both holding the same rope is then the number of ropes is
101
equal to
(A) 101 (B) 100 (C) 51 (D) 50

Q.7 Two fair dice are rolled. Let P(Ai) >0 denotes the probability of the event that the sum of the numbers
appearing on the two dice is divisible by i.

(a) Which one of the following events is most probable?


(A) A3 (B) A4 (C) A5 (D) A6

(b) For which one of the following pairs (i, j) are the events Ai and Aj are independent?
(A) (3, 4) (B) (4, 6) (C) (2, 3) (D) (4, 2)

(c) Number of all possible ordered pairs (i, j) for which the events Ai and Aj are independent.
(A) 6 (B) 12 (C) 13 (D) 25

Q.8 A boy has a collection of blue and green marbles. The number of blue marbles belong to the sets
{2, 3, 4, ..... 13}. If two marbles are chosen simultaneously and at random from his collection, then the
probability that they have different colour is 1 2 . Possible number of blue marbles is :
(A) 2 (B) 3 (C) 6 (D) 10

Q.9 Two eventsAand B are such that the probability that at least one of them occurs is 5/6 and both of them
occurring simultaneously is 1/3. If the probability of not occurrence of B is 1/2 then
(A)Aand B are equally likely (B) A and B are independent
(C) P(A/B) = 2/3 (D) 3 P(A) = 4 P(B)

BANSAL CLASSES Private Ltd. ‘Bansal Tower’, A-10, Road No.-1, I.P.I.A., Kota-05 Page # 44
PROBABILITY

Q.10 The probabilities of events, A  B, A, B & A  B are respectively in A.P. with probability of second
term equal to the common difference. Therefore the events Aand B are
(A) mutuallyexclusive
(B) independent
(C) such that one of them must occur
(D) such that one is twice as likely as the other

Q.11 A student appears for tests I, II and III. The student is successful if he passes either in tests I and II or
1
tests I and III. The probabilities of the student passing in the tests I, II and III are p, q and respectively..
2
1
If the probability that the student is successful is , then
2
(A) p = 1, q = 0
2 1
(B) p = ,q=
3 2
3 2
(C) p = ,q=
5 3
(D) there are infinitely many values of p and q.

Q.12 Three friends A, B and C decides to go to one of the three malls M1, M2 and M3. Mr.A lives near the
mall M1, so the probability that he goes to mall M1 is  5 9  . The probability that Mr. A goes to the
mall M2 is the same as the probability that he goes to the mall M3.
Mr. B lives far from mall M2, so the probability that he goes to mall M2 is  1 7  . The probability that
Mr. B goes to mall M1 is the same as the probability that he goes to mall. M3.
The probability that Mr. C goes to the three malls, mall M1, M2 and M3 are all equal.
If the probability P that atmost 2 out of the 3 friends will meet at the same mall, is expressed as
lowest rational  m n  , then find the sum of the digits in (n – m).

Q.13 In a knockout tournament 2n equally skilled players; S1, S2, ............. S are participating. In each
2n
round players are divided in pair at random and winner from each pair moves in the next round.
1
If S2 reaches the semifinal then the probability that S1 wins the tournament is . Find the value of 'n'.
20

Q.14 An unbiased normal coin is tossed 'n' times.


Let E1 : event that both Heads and Tails are present in 'n' tosses.
E2 : event that the coin shows up Heads atmost once.
Find the value of 'n' for which E1 and E2 are independent.

BANSAL CLASSES Private Ltd. ‘Bansal Tower’, A-10, Road No.-1, I.P.I.A., Kota-05 Page # 45
PROBABILITY

EXERCISE-2
Q.1 There are 2 groups of subjects one of which consists of 5 science subjects & 3 engg. subjects & other
consists of 3 science & 5 engg. subjects .An unbiased die is cast . If the number 3 or 5 turns up a subject
is selected at random from first group, otherwise the subject is selected from 2nd group . Find the
probability that an engg. subject is selected.

Q.2 PlayersAand B alternately toss a biased coin, withAgoing first.Awins ifAtosses a Tail before B tosses
a Head; otherwise B wins. If the probability of a head is p, find the value of p for which the game is fair
to both players.

a b
Q.3 The entries in a two-by-two determinant are integers that are chosen randomlyand independently,,
c d
and, for each entry, the probability that the entry is odd is p. If the probability that the value of the
determinant is even is 1/2, then find the value of p.

a
Q.4 Let an ordinary fair dice is thrown for five times. If P =expressed in lowest form be the probability
b
that the outcome of the fifth throw was already thrown, then find the value of (a + b).

Q.5 A bomber wants to destroy a bridge. Two bombs are sufficient to destroy it.
If four bombs are dropped, what is the probability that it is destroyed, if the chance of a bomb hitting the
target is 0.4.

Q.6 The chance of one event happening is the square of the chance of a 2nd event, but odds against the first
are the cubes of the odds against the 2nd . Find the chances of each. (Assume that both events are neither
sure nor impossible).

Q.7 An aircraft gun can take a maximum of four shots at an enemy’s planemoving awayfrom it.The probability
of hitting the plane at first, second, third & fourth shots are 0.4, 0.3, 0.2 & 0.1 respectively. What is the
probability that the gun hits the plane .

Q.8 In a batch of 10 articles, 4 articles are defective. 6 articles are taken from the batch for inspection.
If more than 2 articles in this batch are defective , the whole batch is rejected Find the probability that
the batch will be rejected.

Q.9 An author writes a good book with a probability of 1/2. If it is good it is published with a probability of
2/3. If it is not, it is published with a probability of 1/4. Find the probability that he will get atleast one
book published if he writes two.
Q.10 A uniform unbiased die is constructed in the shape of a regular tetrahedron with faces numbered 2, 2, 3
and 4 and the score is taken from the face on which the die lands. If two such dice are thrown together,
find the probability of scoring.
(i) exactly 6 on each of 3 successive throws.
(ii) more than 4 on at least one of the three successive throws.

Q.11 Two cards are drawn from a well shuffled pack of 52 cards. Find the probability that one of them
is a red card & the other is a queen.
BANSAL CLASSES Private Ltd. ‘Bansal Tower’, A-10, Road No.-1, I.P.I.A., Kota-05 Page # 46
PROBABILITY

Q.12 Let A & B be two events defined on a sample space . Given P(A) = 0.4 ; P(B) = 0.80 and
P A/ B  = 0.10. Then find ; (i) P AB  & P AB AB  .
1
Q.13 N fair coins are flipped once. The probability that at most 2 of the coins show up as heads is . Find
2
the value of N.
p
Q.14 A coin is tossed 9 times. If that the probability of at least 5 consecutive heads is (where p and q are
q
in their lowest form), then find the value of (p + q).

Q.15 The ratio of the number of trucks along a highway, on which a petrol pump is located, to the number of
cars running along the same highway is 3 : 2. It is known that an average of one truck in thirty trucks and
two cars in fifty cars stop at the petrol pump to be filled up with the fuel. If a vehicle stops at the petrol
pump to be filled up with the fuel, find the probability that it is a car.

Q.16 During a power blackout, 100 persons are arrested on suspect of looting. Each is given a polygraph
test. From past experience it is known that the polygraph is 90% reliable when administered to a guilty
person and 98% reliable when given to some one who is innocent. Suppose that of the 100 persons
taken into custody, only 12 were actually involved in any wrong doing. If the probability that a given
suspect is innocent given that the polygraph says he is guilty is a b where a and b are relatively prime,
find the value of (a + b).

Q.17 A biased coin which comes up heads three times as often as tails is tossed. If it shows heads, a chip is
drawn from urn–I which contains 2 white chips and 5 red chips. If the coin comes up tails, a chip is
drawn from urn–II which contains 7 white and 4 red chips. Given that a red chip was drawn, what is the
probability that the coin came up heads?

Q.18 Abox contains three coins two of them are fair and one two  headed.Acoin is selected at random and
tossed. If the head appears the coin is tossed again, if a tail appears, then another coin is selected from
the remaining coins and tossed.
(i) Find the probability that head appears twice.
(ii) If the same coin is tossed twice, find the probability that it is two headed coin.
(iii) Find the probability that tail appears twice.

Q.19 There are two packs A and B of 52 playing cards. All the four aces from the pack A are removed
whereas from the pack B, one ace, one king, one queen and one jack is removed. One of these two
packs is selected randomly and two cards are drawn simultaneously from it, and found to be a pair
(i.e. both have same rank e.g. two 9's or two king etc). Find the probability that the pack A was
selected.

Q.20 A doctor is called to see a sick child. The doctor knows (prior to the visit) that 90% of the sick children
in that neighbourhood are sick with the flu, denoted by F, while 10% are sick with the measles, denoted
by M.
A well known symptom of measles is a rash, denoted by R. The probability of having a rash for a child
sick with the measles is 0.95. However, occasionally children with the flu also develop a rash, with
conditional probability 0.08.
Upon examining the child, the doctor finds a rash. What is the probability that the child has the measles?
If the probability can be expressed in the form of p q where p, q  N and are in their lowest form,
find (p + q).

BANSAL CLASSES Private Ltd. ‘Bansal Tower’, A-10, Road No.-1, I.P.I.A., Kota-05 Page # 47
PROBABILITY

EXERCISE-3
SECTION-A
(JEE-ADVANCED Previous Year's Questions)

Q1(a) An experiment has 10 equally likely outcomes. LetAand B be two non-empty events of the experiment.
IfAconsists of 4 outcomes, the number of outcomes that B must have so thatAand B are independent,
is
(A) 2, 4 or 8 (B) 3, 6, or 9 (C) 4 or 8 (D) 5 or 10

(b) Consider the system of equations


ax + by = 0, cx + dy = 0, where a, b, c, d  {0, 1}.
3
STATEMENT-1 : The probability that the system of equations has a unique solution is .
8
and
STATEMENT-2 : The probability that the system of equations has a solution is 1.
(A) Statement-1 is True, Statement-2 is True ; statement-2 is a correct explanation for statement-1
(B) Statement-1 is True, Statement-2 is True ; statement-2 is NOT a correct explanation for statement-1
(C) Statement-1 is True, Statement-2 is False
(D) Statement-1 is False, Statement-2 is True [JEE 2008, 3+3]

Comprehension (3 questions)
Q.2 A fair dice is tossed repeatedly until a six is obtained. Let X denote the number of tosses required.

(a) The probability that X = 3 equals


25 25 5 125
(A) (B) (C) (D)
216 36 36 216

(b) The probability that X  3 equals


125 25 5 25
(A) (B) (C) (D)
216 36 36 216

(c) The conditional probability that X  6 given X > 3 equals


125 25 5 25
(A) (B) (C) (D)
216 216 36 36
[JEE 2009, 4+4+4]

Q.3(a) Let  be a complex cube root of unity with   1.Afair dice is thrown three times. If r1, r2 and r3 are the
numbers obtained on the dice, then the probability that r1  r2  r3  0 is
1 1 2 1
(A) (B) (C) (D)
18 9 9 36

BANSAL CLASSES Private Ltd. ‘Bansal Tower’, A-10, Road No.-1, I.P.I.A., Kota-05 Page # 48
PROBABILITY

4 1
(b) Asignal which can be green or red with probability and respectively, is received by stationAand
5 5
3
then transmitted to station B. The probability of each station receiving the signal correctly is . If the
4
signal received at station B is green, then the probability that the original signal was green is
3 6 20 9
(A) (B) (C) (D)
5 7 23 20
[JEE 2010, 3 + 5]
Paragraph for Question Nos. 4 and 5
Let U1 and U2 be two urns such that U1 contains 3 white and 2 red balls, and U2 contains only 1 white
ball. A fair coin is tossed. If head appears then 1 ball is drawn at random from U1 and put into U2.
However, if tail appears then 2 balls are drawn at random from U1 and put into U2. Now 1 ball is drawn
at random from U2.
Q.4 The probability of the drawn ball from U2 being white is
13 23 19 11
(A) (B) (C) (D)
30 30 30 30
Q.5 Given that the drawn ball from U2 is white, the probability that head appeared on the coin is
17 11 15 12
(A) (B) (C) (D)
23 23 23 23
[JEE 2011, 3+3]

1
Q.6 Let E and F be two independent events. The probability that exactly one of them occurs is and the
25
2
probability of none of them occurring is . If P(T) denotes the probability of occurrence of the event
25
T, then
 3  2
(A) P(E) = , P(F) = (B) P(E) = , P(F) =
5 5 5 5
2 1 3 4
(C) P(E) = , P(F) = (D) P(E) = , P(F) =
5 5 5 5
[JEE 2011, 4]

Q.7 Four fair dice D1, D2, D3, and D4 each having six faces numbered 1, 2, 3, 4, 5 and 6 are rolled
simultaneously. The probability that D4 shows a number appearing on one of D1, D2 and D3 is
91 108 125 127
(A) (B) (C) (D)
216 216 216 216
[JEE 2012, 3]

BANSAL CLASSES Private Ltd. ‘Bansal Tower’, A-10, Road No.-1, I.P.I.A., Kota-05 Page # 49
PROBABILITY

Q.8 A ship is fitted with three engines E1, E2 and E3. The engines function independently of each other with
1 1 1
respective probabilities , and . For the ship to be operational at least two of its engines must
2 4 4
function. Let X denotes the event that the ship is operational and let X1, X2 and X3 denotes respectively
the events that the engines E1, E2 and E3 are functioning. Which of the following is(are) true?


(A) P X1c | X  3
16
(B) P [Exactlytwo engines of the ship are functioning | X] =
7
8
5 7
(C) P [X | X2] = (D) P [X | X1] =
16 16
[JEE 2012, 4]
1 3 1 1
Q.9 Four persons independently solve a certain problem correctly with probabilities , , , . Then the
2 4 4 8
probability that the problem is solved correctly by atleast one of them is
235 21 3 253
(A) (B) (C) (D)
256 256 256 256
[JEE (Advanced) 2013, 2]

Q.10 Of the three independent events E1, E2 and E3, the probability that only E1 occurs is , only E2 occurs
is  and only E3 occurs is . Let the probability p that none of events E1, E2 or E3 occurs satisfy the
equations ( –2) p =  and ( – 3) p = 2 .
All the given probabilities are assumed to lie in the interval (0, 1).
Probability of occurence of E1
Then Probability of occurence of E  [JEE (Advanced) 2013, 4]
3

Paragraph for Questions 11 & 12

A box B1 contains 1 white ball, 3 red balls and 2 black balls. Another box B2 contains 2 white balls,
3 red balls and 4 black balls. Athird box B3 contains 3 white balls, 4 red balls and 5 black balls.
Q.11 If 2 balls are drawn (without replacement) from a randomly selected box and one of the balls is white
and the other ball is red, the probability that these 2 balls are drawn from box B2 is
116 126 65 55
(A) (B) (C) (D)
181 181 181 181

Q.12 If 1 ball is drawn from each of the boxes B1, B2 and B3, the probability that all 3 drawn balls are of the
same colour is
82 90 558 566
(A) (B) (C) (D)
648 648 648 648
[JEE (Advanced) 2013, 3 + 3]

Q.13 Three boys and two girls stand in a queue. The probability, that the number of boys ahead of every girl
is at least one more than the number of girls ahead of her, is
(A) 1/2 (B) 1/3 (C) 2/3 (D) 3/4
[JEE (Advanced) 2014, 3]

BANSAL CLASSES Private Ltd. ‘Bansal Tower’, A-10, Road No.-1, I.P.I.A., Kota-05 Page # 50
PROBABILITY

Paragraph for Questions 14 and 15


Box 1 contains three cards bearing numbers 1, 2, 3; box 2 contains five cards bearing numbers 1, 2, 3,
4, 5; and box 3 contains seven cards bearing numbers 1, 2, 3, 4, 5, 6, 7. A card is drawn from each of
the boxes. Let xi be the number on the card drawn from the ith box, i = 1, 2, 3.
Q.14 The probability that x1 + x2 + x3 is odd, is
29 53 57 1
(A) (B) (C) (D)
105 105 105 2
Q.15 The probability that x1, x2, x3 are in an arithmetic progression, is
9 10 11 7
(A) (B) (C) (D)
105 105 105 105
[JEE (Advanced) 2014, 3]

Q.16 The minimum number of times a fair coin needs to be tossed, so that the probability of getting at least two
heads is at least 0.96 is [JEE (Advanced) 2015, 4]

Paragraph for Questions 17 and 18


Let n1 and n2 be the number of red and black balls, respectively, in box I. Let n3 and n4 be the number
of red and black balls, respectively, in box II.
Q.17 One of the two boxes, box I and box II, was selected at random and a ball was drawn randomly out of
1
this box. the ball was found to be red. If the probability that this red ball was drawn from box II is ,
3
then the correct option(s) with the possible values of n1, n2, n3 and n4 is(are)
(A) n1 = 3, n2 = 3, n3 = 5, n4 = 15 (B) n1 = 3, n2 = 6, n3 = 10, n4 = 50
(C) n1 = 8, n2 = 6, n3 = 5, n4 = 20 (D) n1 = 6, n2 = 12, n3 = 5, n4 = 20

Q.18 A ball is drawn at random from box I and transferred to box II. If the probability of drawing a red ball
1
from box I, after this transfer, is , then the correct option(s) with the possible values of n1 and n2
3
is(are)
(A) n1 = 4 and n2 = 6 (B) n1 = 2 and n2 = 3
(C) n1 = 10 and n2 = 20 (D) n1 = 3 and n2 = 6
[JEE (Advanced) 2015, 4 + 4]

Q.19 A computer producing factory has only two plants T1 and T2. Plant T1 produces 20% and plant T2
produces 80% of the total computers produced. 7% of computers produced in the factory turn out to be
defective. It is known that
P (computer turns out to be defective given that it is produced in plant T1)
= 10 P (computer turns out to be defective given that it is produced in plant T2),
where P(E) denotes the probability of an event E. A computer produced in the factory is randomly
selected and it does not turn out to be defective. Then the probability that it is produced in plant T2 is
36 47 78 75
(A) (B) (C) (D)
73 79 93 83
[JEE (Advanced) 2016, 3]

BANSAL CLASSES Private Ltd. ‘Bansal Tower’, A-10, Road No.-1, I.P.I.A., Kota-05 Page # 51
PROBABILITY

Paragraph for Questions 20 and 21


Football teams T1 and T2 have to play two games against each other. It is assumed that the outcomes of
the two games are independent. The probabilities of T1 winning, drawing and losing a game against T2
1 1 1
are , and , respectively. Each team gets 3 points for a win, 1 point for a draw and 0 point for a
2 6 3
loss in a game. Let X and Y denote the total points scored by teams T1 and T2 respectively, after two
games.
Q.20 P(X > Y) is
1 5 1 7
(A) (B) (C) (D)
4 12 2 12
Q.21 P(X = Y) is
11 1 13 1
(A) (B) (C) (D) .
36 3 36 2
[JEE (Advanced) 2016, 3+3]
1 1 2
Q.22 Let X and Y be two events such that P(X) = , P(X|Y) = and P(Y|X) = . Then
3 2 5
1 4 1 2
(A) P(X  Y) = (B) P(Y) = (C) P(X'|Y) = (D) P (X  Y) =
5 15 2 5
[JEE (Advanced) 2017, 4]
Q.23 Three randomly chosen non-negative integers x, y and z are found to satisfy the equation x + y + z = 10.
Then the probability that z is even, is
5 1 6 36
(A) (B) (C) (D)
11 2 11 55
[JEE (Advanced) 2017, 4]
Paragraph for Question no. 24 & 25

There are five students S1, S2, S3, S4 and S5 in a music class and for them there are five seats R1, R2, R3,
R4 and R5 arranged in a row, where initially the seat Ri is allotted to the student Si, i = 1, 2, 3, 4, 5. But,
on the examination day, the five students are randomly allotted the five seats.

Q.24 The probability that, on the examination day, the student S1 get the previously allotted seat R1, and
NONE of the remaining students gets the seat previously allotted to him/her is
3 1 7 1
(A) (B) (C) (D)
40 8 40 5

Q.25 For i = 1, 2, 3, 4, let Ti denote the event that the student Si and Si+1 do NOT sit adjacent to each other
on the day of the examination. Then, the probability of the event T1  T2  T3  T4 is
1 1 7 1
(A) (B) (C) (D)
15 10 60 5
[JEE (Advanced) 2018, 3+3]

BANSAL CLASSES Private Ltd. ‘Bansal Tower’, A-10, Road No.-1, I.P.I.A., Kota-05 Page # 52
PROBABILITY

SECTION-B
(JEE-MAIN Previous Year's Questions)
Q.1 Adie is thrown. LetAbe the event that the number obtained is greater than 3. Let B be the event that the
number obtained is less than 5. Then P(A  B) is [AIEEE 2008]
2 3
(1) 0 (2) 1 (3) (4)
5 5

1 1 2
Q.2 It is given that the events A and B are such that P(A) = , P(A|B) = and P(B|A) = . Then P(B) is
4 2 3

1 2 1 1
(1) (2) (3) (4) [AIEEE 2008]
3 3 2 6

In a binomial distribution B  n, p   , if the probability of at least one success is greater than or equal to
1
Q.3
 4

9
, then n is greater than : [AIEEE 2009]
10

1 9 4 1
(1) 4 3 (2) 4 3 (3) 4 3 (4) 4 3
log10  log10 log10  log10 log10  log10 log10  log10

Q.4 One ticket is selected at random from 50 tickets numbered 00, 01, 02, ......., 49. Then the probability
that the sum of the digits on the selected ticket is 8, given that the product of these digits is zero, equals:
1 5 1 1
(1) (2) (3) (4) [AIEEE 2009]
7 14 50 14

Q.5 An urn contains nine balls of which three are red, four are blue and two are green. Three balls are drawn
at random without replacement from the urn. The probabilitythat the three balls have different colours, is
1 2 1 2
(1) (2) (3) (4) [AIEEE 2010]
3 7 21 23

Q.6 Four numbers are chosen at random (without replacement) from the set {1, 2, 3,...., 20}.
Statement-1 : The probability that the chosen numbers when arranged in some order will form anAP,
1
is .
85
Statement-2 : If the four chosen numbers from anAP, then the set of all possible values of common
difference is {± 1, ± 2, ± 3, ± 4, ± 5}.
(1) Statement-1 is true, statement-2 is true and statement-2 is correct explanation for statement-1.
(2) Statement-1 is true, statement-2 is true and statement-2 is NOT the correct explanation for statement-1.
(3) Statement-1 is true, statement-2 is false.
(4) Statement-1 is false, statement-2 is true. [AIEEE 2010]

BANSAL CLASSES Private Ltd. ‘Bansal Tower’, A-10, Road No.-1, I.P.I.A., Kota-05 Page # 53
PROBABILITY

Q.7 Consider 5 independent Bernoulli's trials each with probability of success p. If the probability of at least
31
one failure is greater than or equal to , then p lies in the interval [AIEEE 2011]
32

 1 3  3 11   1  11 
(1)  ,  (2)  ,  (3) 0, (4)  , 1
 2 4  4 12   2   12 

Q.8 If C and D are two events such that C  D and P(D)  0, then the correct statement among the following
is
(1) P(C | D) = P(C) (2) P(C | D)  P(C)
P ( D)
(3) P(C | D) < P(C) (4) P(C | D) = [AIEEE 2011]
P ( C)

Q.9 Three numbers are chosen at random without replacement from {1, 2, 3, ........, 8}. The probability that
their minimum is 3, given that their maximum is 6, is
1 2 3 1
(1) (2) (3) (4) [AIEEE 2012]
4 5 8 5

Q.10 A multiplechoice examination has 5questions.Each questionhas three alternative answers ofwhich exactly
one is correct. The probability that a student will get 4 or more correct answers just by guessing is
13 11 10 17
(1) (2) (3) (4) [JEE (Main) 2013]
35 35 35 35

Q.11 
Let A and B be two events such that P A  B   1
6
1 1
, PA  B  and P( A )  , where A stands
4 4
for the complement of the event A. Then the events Aand B are
(1) independent and equally likely (2) mutuallyexclusive and independent
(3) equally likely but not independent (4) independent but not equally likely
[JEE (Main) 2014]

Q.12 If 12 identical balls are to be placed in 3 identical boxes, then the probability that one of the boxes
contains exactly3 balls is
12 11 11 10
1 1 55  2  2
(1) 220  (2) 22  (3)   (4) 55 
 3 3 3 3 3
[JEE (Main) 2015]

BANSAL CLASSES Private Ltd. ‘Bansal Tower’, A-10, Road No.-1, I.P.I.A., Kota-05 Page # 54
PROBABILITY

Q.13 Let two fair six-faced diceAand B be thrown simultaneously. If E1 is the event that dieAshows up four,
E2 is the event that die B shows up two and E3 is the event that the sum of numbers on both dice is odd,
then which of the following statements is NOT true?
(1) E1, E2 and E3 are independent. (2) E1 and E2 are independent.
(3) E2 and E3 are independent. (4) E1 and E3 are independent. [JEE (Main) 2016]

Q.14 For three events A, B and C,


P (Exactly one of A or B occurs) = P (Exactly one of B or C occurs)
1 1
= P(Exactly one of C or A occurs) = and P (All the three events occur simultaneously) = .
4 16
Then the probability that at least one of the events occurs, is
7 7 7 3
(1) (2) (3) (4)
32 16 64 16
[JEE (Main) 2017]

Q.15 If two different numbers are taken from the set {0, 1, 2, 3, ……, 10}; then the probability that their sum
as well as absolute difference are both multiple of 4, is
6 12 14 7
(1) (2) (3) (4)
55 55 45 55
[JEE (Main) 2017]

Q.16 A box contains 15 green and 10 yellow balls. If 10 balls are randomly drawn, one-by-one, with
replacement, then the variance of the number of green balls drawn is
12 6
(1) (2) 6 (3) 4 (4)
5 25
[JEE (Main) 2017]

Q.17 A bag contains 4 red and 6 black balls. A ball is drawn at random from the bag, its colour is observed
and this ball along with two additional balls of the same colour are returned to the bag. If now a ball is
drawn at random from the bag, then the probability that this drawn ball is red, is :
1 3 3 2
(1) (2) (3) (4)
5 4 10 5
[JEE (Main) 2018]

BANSAL CLASSES Private Ltd. ‘Bansal Tower’, A-10, Road No.-1, I.P.I.A., Kota-05 Page # 55
PROBABILITY

EXERCISE-4
SECTION-A
(CBSE Previous Year's Questions)
Q.1 A and B throw a pair of dice turn by turn. The first to throw 9 is awarded a prize. If A starts the game,
9
show that the probability ofAgetting the prize is . [CBSE AI 2008]
17

Q.2 An insurance company insured 2000 scooter drivers, 4000 car drivers and 6000 truck drivers. The
probability of an accident involving a scooter driver, car driver and a truck driver is 0.01, 0.03 and 0.15
respectively. One of the insured person meets with an accident. What is the probability that he is a
scooter driver ? [CBSE Delhi 2008, 2003 C, 2002, 2000]
Q.3 A die is thrown again and again until three sixes are obtained. Find the probability of obtaining the third
six in the sixth throw of the die. [CBSE Delhi 2009]
1 2
Q.4 The probability that A hits a target is and the probability that B hits it is . If each one of A and B
3 5
shoots at the target, what is the probability that
(i) the target is hit ? (ii) exactly one of them hits the target ?
[CBSE (F) 2009]
Q.5 Two groups are competing for the position on the Board of directors of a corporation. The probabilities
that the first and the second group will win are 0.6 and 0.4 respectively. Further, if the first group wins,
the probability of introducing a new product is 0.7 and the corresponding probability is 0.3, if the second
group wins. Find the probability that the new product was introduced by the second group.
[CBSE Delhi 2009]
Q.6 From a lot of 30 bulbs which includes 6 defective, a sample of 4 balls is drawn at random with replacement.
Find the mean and variance of the number of defective bulbs. [CBSE (F) 2009]
Q.7 Three bags contain balls as shown in the table below :

Bag Number of white balls Number of Black balls Number of Re d balls


I 1 2 3
II 2 1 1
III 4 3 2

A bag is chosen at random and two balls are drawn from it. They happen to be white and red. What is
the probability that they come from the III bag ? [CBSE Delhi 2009]

BANSAL CLASSES Private Ltd. ‘Bansal Tower’, A-10, Road No.-1, I.P.I.A., Kota-05 Page # 56
PROBABILITY

Q.8 Coloured balls are distributed in three bags as shown in the following table :
Colour of the Ball
Bag
Black White Red
I 1 2 3

II 2 4 1

III 4 5 3
A bag is selected at random and then two balls are randomly drawn from the selected bag. They happen
to be black and red. What is the probability that they came from bag I ? [CBSE (AI) 2009]
Q.9 A family has 2 children. Find the probability that both are boys, if it is known that
(i) at least one of the children is a boy.
(ii) the elder child is a boy [CBSE (AI) 2010]

Q.10 There are two Bags, Bag I and Bag II. Bag I contains 4 white and 3 red balls while another Bag II
contains 3 white and 7 red balls. One ball is drawn at random from one of the bags and it is found to be
white. Find the probability that it was drawn from Bag I. [CBSE Delhi 2010]

Q.11 Abag contains 4 balls. Two balls are drawn at random and are found to be white. What is the probability
that all balls are white? [CBSE (A1) 2010]

Q.12 An urn contains 4 white and 3 red balls. Let X be the number of red balls in a random draw of three balls.
Find the mean and variance of X. [CBSE (F) 2010]

Q.13 On a multiple choice examination with three possible answer (out of which only one to correct) for each
of the give questions, what is the probability that a candidate would get four or more correct answer just
by guessing ? [CBSE Delhi 2010, (AI) 2009]

Q.14 Bag I contains 3 red and 4 black balls while anothe Bag II contains 5 red and 6 black balls. One ball is
drawn at random from one of the bags and it is found to be red. Find the probability that it was drawn
from Bag II. [CBSE Delhi 2011]

Q.15 A random variable X has the following probability distribution

X 0 1 2 3 4 5 6 7
P( x ) 0 k 2k 2k 3k k 2 2k 2 7k 2  k

Determine :
(i) k (ii) P(X < 3) (iii) P(X > 6) (iv) P(0 < X < 3)
[CBSE (AI) 2011]

BANSAL CLASSES Private Ltd. ‘Bansal Tower’, A-10, Road No.-1, I.P.I.A., Kota-05 Page # 57
PROBABILITY

Q.16 A factory has two machines Aand B. Past record shows that machine Aproduced 60% of the items of
output and machine B produced 40% of the items. Further, 2% of the items produced by machineAand
1% produced by machine B were defective.All the items are put into one stockpile and then one item is
chosen at random from this and is found to be defecitve. What is the probability that it was produced by
machine B ? [CBSE (F) 2011]

Q.17 A man is known to speak truth 3 out of 4 times. He throws a die and reports that it is a six. Find the
probaility that it is actually a six. [CBSE Delhi 2011, Delhi 2005, (AI) 2003 C]

Q.18 There are three coins. one is two headed coin (having head on both face), another is a based coin that
comes up tail 25% of the times and third is an unbaised coin. One of the three coins is chosen at random
and tossed, it shows heads, what is the probability that is was two headed coin ?
[CBSE Delhi (F) 2011, 2009]

Q.19 Two cards are drawn simultaneously (without replacement) from a well-shuffled pack of 52 cards. Find
the mean and variance of the number of red cards.
[CBSE 2012]

Q.20 Suppose a girl throws a die. If she gets a 5 or 6, she tosses a coin 3 times and notes the number of heads.
If she gets 1, 2, 3 or 4 she tosses a coin once and notes whether a head or tail is obtained. If she obtained
exactly one head, what is the probability that she threw 1, 2, 3 or 4 with the die? [CBSE 2012]

3 5
Q.21 The probabilities of two students A and B coming to the school in time are and respectively..
7 7
Assuming that the events, 'A coming in time' and 'B coming in time' are independent, find the probability
of only one of them coming to the school in time.
Write at least one advantage of coming to school in time. [CBSE 2013]

Q.22 In a hockey match, both teams A and B scored same number of goals up to the end of the game, so to
decide the winner, the referee asked both the captains to throw a die alternately and decided that the
team, whose captain gets a six first, will be declared the winner. If the captain of team Awas asked to
start, find their respective probabilities of winning the match and state whether the decision of the referee
was fair or not. [CBSE 2013]

Q.23 An experiment succeeds thrice as often as it fails. Find the probabilitythat in the next five trials, there will
be at least 3 successes. [CBSE 2014]

Q.24 A man takes a step forward with probability 0.4 and backward with probability 0.6. Find the probability
that at the end of 5 steps, he is one step away from the starting point. [CBSE 2015]

BANSAL CLASSES Private Ltd. ‘Bansal Tower’, A-10, Road No.-1, I.P.I.A., Kota-05 Page # 58
PROBABILITY

Q.25 Suppose a girl throws a die. If she gets a 1 or 2, she tosses a coin three times and notes the number of
'tails'. If she gets 3, 4, 5 or 6, she tosses a coin once and notes whether a 'head' or 'tail' is obtained. If she
obtained exactly one 'tail', what is the probability that she threw 3, 4, 5 or 6 with the die?
[CBSE 2015]

Q.26 An urn contains 5 red and 2 black balls. Two balls are randomly drawn, without replacement. Let X
represent the number of black balls drawn. What are the possible values of X? Is X a random variable?
If yes, find the mean and variance of X. [CBSE 2015]

Q.27 Abag X contains 4 white ball and 2 black balls, while another bag Ycontains 3 white balls and 3 black
balls. Two balls are drawn (without replacement) at random from one of the bags and where found to be
one white and one black. Find the probability that the ball were drawn from bag Y. [CBSE 2016]

Q.28 A and B throw a pair of dice alternately, till one of them gets a total of 10 and wins the game. Find their
respective probabilities of winning, if A starts first. [CBSE 2016]

Q.29 Three numbers are selected at random (without replacement) from first six positive integers. Let X
denote the largest of the three numbers obtained. Find the probability distribution of X.Also, find the
mean and variance of the distribution. [CBSE 2016]

Q.30 Adie, whose faces are marked 1, 2, 3 in red and 4, 5, 6 in green, is tossed. Let Abe the event "number
obtained is even" and B be the event "number obtained is red". Find if A and B are independent
events. [CBSE 2017]

Q.31 Of the students in a school, it is known that 30% have 100% attendance and 70% students are irregular.
Previous year results report that 70% of all students who have 100% attendance attainAgrade and 10%
irregular students attainAgrade in their annual examination.At the end of the year, one student is chosen
at random from the school and he was found to have anAgrade. What is the probability that the student
has 100% attendance? Is regularity required only in school? Justify your answer. [CBSE 2017]

Q.32 Ablack and a red die are rolled together. Find the conditional probability of obtaining the sum 8, given
that the red die resulted in a number less than 4. [CBSE 2018]
Q.33 Suppose a girl throws a die. If she gets 1 or 2, she tosses a coin three times and notes the number of tails.
If she gets 3, 4, 5 or 6, she tosses a coin once and notes whether a 'head' or 'tail' is obtained. If she
obtained exactly one 'tail', what is the probability that she threw 3, 4, 5 or 6 with the die?
[CBSE 2018]

Q.34 Two numbers are selected at random (without replacement) from the first five positive integers. Let X
denote the larger of the two numbers obtained. Find the mean and variance of X. [CBSE 2018]

BANSAL CLASSES Private Ltd. ‘Bansal Tower’, A-10, Road No.-1, I.P.I.A., Kota-05 Page # 59
PROBABILITY

SECTION-B
(Potential Problems Based on CBSE)
Q.1 Write the probability of an even prime number on each die, when a pair of dice is rolled.

Q.2 Afair coin and unbiased die are tossed. LetAbe the event 'head appears on the coin' and B be the event
'3 on the die'. Check whether Aand B are independent events or not.

Q.3 The probability distribution of X is

X 0 1 2 3
P ( X ) 0 .2 K K 2 K

Write the value of K.

Q.4 A four digit number is formed using the digits 1, 2, 3, 5 with no repetitions. Find the probability that the
number is divisible by 5.

Q.5 Assume that each born child is equally likely to be a boy or a girl. If a family has two children, what is the
conditional probability that both are girls given that (i) the youngest is a girl, (ii) at least one is a girl ?
1 1 1
Q.6 A problem in mathematics is given to 3 students whose chances of solving it are , , . What is the
2 3 4
probability that the (i) problem is solved (ii) exactly one of them will solve it ?

Q.7 In a game, a man wins a rupee for a six and loses a rupee for any other number when a fair die is thrown.
The man decided to throw a die thirce but to quit as and when he gets a six. Find the expected value of
the amount he wins/loses.

Q.8 A speaks truth in 60% of the cases and B in 90% of the cases. In what percentage of cases are they
likely to contradict each other in stating the same fact ?

Q.9 Two cards are drawn without replacement from a well shuffled pack of 52 cards. Find the probability
that one is a spade and other is a queen of red colour.

Q.10 A coin is tossed 4 times. Find the mean and variance of the probability distribution of the number of
heads.

Q.11 The mean and variance of a binomial distribution are 12 and 3 respectively. Findthe probabilitydistribution.

Q.12 A pair of dice is thrown 4 times. If gettinga doublet is considered a success, find the probabilitydistribution
of number of successes.

Q.13 Three machines E1, E2, E3 in a certain factory produce 50%, 25% and 25% respectively, of the total
daily output of electric tubes. It is known that 4% of the tube produced on each of machines E1 and E2
are defective and that 5% of those produced on E3, are defective. If one tube is picked up at random
from a day's production, calculate the probability that it is defective.

BANSAL CLASSES Private Ltd. ‘Bansal Tower’, A-10, Road No.-1, I.P.I.A., Kota-05 Page # 60
PROBABILITY

3
Q.14 In answering a question on a multiple choice test, a student either knows the answer or gusses. Let be
4
1
the probability that he knows the answer and be the probability that he guesses. Assuming that a
4
student who guesses at the answer will be correct with probability, what is the probabilitythat the student
knows the answer given that he answered it correctly ?

Q.15 A card from a pack of 52 cards is lost. From the remaining cards of the pack, two cards are drawn at
random and are found to both clubs. Find the probability of the lost card being the clubs.

Q.16 Amotorcycle manufacturing factory has two plants, X andY. Plant X manufactures 70% of motorcycles
and plant Y manufactures 30%.At plant X, 80% of the motorcycles are rated of standard quality and at
plant Y, 90% of the motorcycles are rated of standard quality.Amotorcycle is picked up at random and
is found to be of standard quality. What is the probability that it has come from plant Y ?

Q.17 Let X denote the number of colleges, where you will apply after your results and P(X = x) denotes your
probability of getting admission in X number of collges. It is given that

kx if x  0 or 1

P(X = x) = 2kx if x2 k is positive constant.
k (5  x ) if x  3 or 4

(a) Find the value of k.


(b) What is the probability that you will get admission in exactly two collges ?
(c) Find the mean and variance of probability distribution.

Q.18 Aletter is known to have come either from TATANAGAR or CALCUTTA. On the envelope just two
consecutive letter TAare visible. What is the probability that letter has come from (i) CALCUTTA(ii)
TATANAGAR.

Q.19 Two bags A and B contain 4 white, 3 black balls and 2 white, 2 black balls respectively. From bag A,
two balls are transferred to bag B. Find the probability of drawing
(a) 2 white balls from bag B.
(b) 2 black balls from bag B.
(c) 1 white and 1 black ball from bag B.

Q.20 There are three urnsA, B and C. UrnAcontains 4 white balls and 5 blue balls. Urn B contains 4 white balls
and 3 blue balls. Urn C contains 2 white balls and 4 blue balls. One ball is drawn from each of these urns.
What is the probability that out of these three balls drawn, two are white balls and one is a blue ball?

BANSAL CLASSES Private Ltd. ‘Bansal Tower’, A-10, Road No.-1, I.P.I.A., Kota-05 Page # 61
PROBABILITY

EXERCISE-5 (Rank Booster)


1
Q.1 Consider 4 independent trials in which an event Aoccurs with probability . The event B will occur
3
with probability 1 if the eventA occurs atleast twice, it can not occur if the eventA does not occur and
1
it occurs with a probability if the event A occurs once. If the probability P of the occurrence of
2
m
event B can be expressed as , find the least value of (m + n), where m, n  N.
n

Q.2 A person flips 4 fair coins and discards those which turn up tails. He again flips the remaining coin and
then discards those which turn up tails. Find the probability that he discards atleast 3 coins.

Q.3 Three shots are fired independently at a target in succession. The probabilities that the target is hit in the
1 2 3
first shot is , in the second and in the third shot is . In case of exactly one hit , the probability of
2 3 4
1 7
destroying the target is and in the case of exactly two hits, and in the case of three hits is 1.0.
3 11
Find the probability of destroying the target in three shots.
Q.4 16 players take part in a tennis tournament. The order of the matches is chosen at random. There is
always a player better than another one, the better wins. Find
(a) The probability that all the 4 best players reach the semifinals.
(b) The probabilitythat the sixth best reaches the semifinals.

Q.5 A certain drug, manufactured by a Company is tested chemically for its toxic nature. Let the event "THE
DRUG IS TOXIC" be denoted by H and the event "THE CHEMICAL TEST REVEALS THAT THE DRUG IS TOXIC"

be denoted by S. Let P(H) = a, P S / H = P S / H  = 1  a. Then show that the probability that the
drug is not toxic given that the chemical test reveals that it is toxic, is free from 'a'.

Q.6 If two whole numbers x and y are randomly selected from the set of natural numbers, then find the
probability that x3 + y3 is divisible by 8.

a2
Q.7 A hunter’s chance of shooting an animal at a distance 'r' is 2 (r > a). He fires when r = 2a and
r
if he misses he reloads & fires when r = 3a, 4a, ..... If he misses at a distance ‘na’, the animal escapes.
Find the odds against the hunter.

Q.8 In an hotel International, 40% English and 60%Americans people are staying. English andAmerican
spellings for the same word are 'RIGOUR' and 'RIGOR', respectively.Arandomly chosen man staying
at a Nigerian hotel writes this word, and a letter taken at random from his spelling was found to be a
vowel. Then the probability that the writer is an Englishman, can be expressed as p q  , p, q  N,
find the least value of (p + q).
BANSAL CLASSES Private Ltd. ‘Bansal Tower’, A-10, Road No.-1, I.P.I.A., Kota-05 Page # 62
PROBABILITY

Q.9 In a tournament, team X, plays with each of the 6 other teams once. For each match the probabilities of
a win, draw and loss are equal. Find the probability that the team X, finishes with more wins than losses.

Q.10 With respect to a particular question on a multiple choice test (having 4 alternatives with only 1 correct)
a student knows the answer and therefore can eliminate 3 of the 4 choices from consideration with
2 1
probability , can eliminate 2 of the 4 choices from consideration with probability , can eliminate
3 6

1 1
1 choice from consideration with probability , and can eliminate none with probability . If the
9 18
student knows the answer, he answers correctly, otherwise he guesses from among the choices not
eliminated.
a
If the answer given by the student was found correct, then the probability that he knew the answer is
b
where a and b are relatively prime. Find the value of (a + b).

BANSAL CLASSES Private Ltd. ‘Bansal Tower’, A-10, Road No.-1, I.P.I.A., Kota-05 Page # 63
PROBABILITY

EXERCISE-1
SPECIAL DPP-1
Q.1 A Q.2 A Q.3 A Q.4 D Q.5 A
Q.6 B Q.7 A Q.8 A Q.9 C Q.10 B
Q.11 B Q.12 23/168 Q.13 5 : 1
Q.14 (A) R, (B) S, (C) Q, (D) P; (E) R

SPECIAL DPP-2

Q.1 (i) 0.18, (ii) 0.12, (iii) 0.42, (iv) 0.28, (v) 0.72 Q.2 2/3
Q.3 3/4, 1/4; 15/16 Q.4 3/5 Q.5 D Q.6 A
Q.7 B Q.8 B Q.9 C Q.10 A Q.11 B
Q.12 BCD Q.13 BC Q.14 12/25

SPECIAL DPP-3

Q.1 D Q.2 D Q.3 A Q.4 C Q.5 D


Q.6 B Q.7 A Q.8 C Q.9 D Q.10 C
Q.11 D Q.12 A Q.13 ACD Q.14 AD
Q.15 ABCD Q.16 155 Q.17 17/105 Q.18 209/343

SPECIAL DPP-4

Q.1 C Q.2 D Q.3 C Q.4 C Q.5 A


Q.6 B Q.7 C Q.8 B Q.9 C Q.10 C
Q.11 A Q.12 B Q.13 B Q.14 D Q.15 BCD
Q.16 ABD Q.17 13 to 5 Q.18 0190

SPECIAL DPP-5
Q.1 B Q.2 B Q.3 B Q.4 C Q.5 C
Q.6 A Q.7 B Q.8 C Q.9 D Q.10 B
Q.11 B Q.12 A Q.13 D Q.14 C Q.15 B
Q.16 D Q.17 B Q.18 C Q.19 ABC Q.20 ACD
Q.21 ABC

BANSAL CLASSES Private Ltd. ‘Bansal Tower’, A-10, Road No.-1, I.P.I.A., Kota-05 Page # 64
PROBABILITY

SPECIAL DPP-6
Q.1 C Q.2 C Q.3 C Q.4 D Q.5 A
Q.6 C Q.7 B Q.8 B Q.9 D Q.10 A
Q.11 BC Q.12 CD Q.13 CD Q.14 27

SPECIAL DPP-7
Q.1 A Q.2 B Q.3 D Q.4 D Q.5 C
Q.6 C Q.7 (a) A (b) C (c) D Q.8 BCD Q.9 BCD
Q.10 AD Q.11 ABCD Q.12 5 Q.13 4 Q.14 3

EXERCISE-2

5 1 2
Q.1 13/24 Q.2 Q.3 Q.4 1967
2 2

328 1 1
Q.5 Q.6 , Q.7 0.6976 Q.8 19/42
625 9 3

125 63
Q.9 407/576 Q.10 (i) 3 ; (ii) Q.11 101/1326
16 64
Q.12 (i) 0.82, (ii) 0.76 Q.13 5 Q.14 35 Q.15 4/9
12
Q.16 179 Q.17 165/193 Q.18 1/2, 1/2, 1/12 Q.19
23
Q.20 262

EXERCISE-3
SECTION-A
Q.1 (a) D, (b) B Q.2 (a) A, (b) B, (c) D Q.3 (a) C; (b) C
Q.4 B Q.5 D Q.6 AD Q.7 A Q.8 BD
Q.9 A Q.10 6 Q.11 D Q.12 A Q.13 A
Q.14 B Q.15 C Q.16 8 Q.17 A, B Q.18 C, D
Q.19 C Q.20 B Q.21 C Q.22 BC Q.23 C
Q.24 A Q.25 C
SECTION-B
Q.1 2 Q.2 1 Q.3 4 Q.4 4 Q.5 2
Q.6 3 Q.7 3 Q.8 2 Q.9 4 Q.10 2
Q.11 4 Q.12 3 Q.13 1 Q.14 2 Q.15 1
Q.16 1 Q.17 4

BANSAL CLASSES Private Ltd. ‘Bansal Tower’, A-10, Road No.-1, I.P.I.A., Kota-05 Page # 65
PROBABILITY

EXERCISE-4
SECTION-A

625
Q.1 9/17 Q.2 1/52 Q.3 Q.4 (i) 3/5 (ii) 7/15
3  65
Q.5 2/9 Q.6 4/5, 16/25 Q.7 5/17 Q.8 231/551

9 24
Q.9 (i) 1/3 (ii) 1/2 Q.10 40/61 Q.11 3/5 Q.12 ,
7 49
Q.13 11/243 Q.14 35/68 Q.15 (i) 1/10 (ii) 3/10 (iii) 17/100 (iv) 3/10

25
Q.16 1/4 Q.17 3/8 Q.18 4/9 Q.19 Mean = 1, variance =
51

8 26 6 5 459
Q.10 Q.21 Q.22  ,  Q.23
11 49 11 11 512

65 11 8
Q.24 C5 Q.25
510 11
Q.26 Possible value of x are 0, 1, 2
yes, x is a random variable since its domain is the sample space of the random experiment
mean = 4/7
Variance = 50/147

9 12 11 21 63
Q.27 Q.28 P( 'A' wins) = , P ('B' wins) = Q.29 ,
17 23 23 4 80

63 1
Q.30 Not independent Q.31 Q.32 Q.33 8/11
112 9
Q.34 4, 1

SECTION-B

1 1 1
Q.1   Q.2 A and B are independent events Q.3 k = 0.2
6 6 36
Q.4 1/4 Q.5 1/3 Q.6 11/24
Q.7 11/216 Q.8 21/50 Q.9 1/51
16
1 3
Q.10 2, 1 Q.11 Binomial distribution is (q + p)n =   
4 4

BANSAL CLASSES Private Ltd. ‘Bansal Tower’, A-10, Road No.-1, I.P.I.A., Kota-05 Page # 66
PROBABILITY

X  xi 0 1 2 3 4

Q.12 625 125 25 5 1 Q.13 0.0425


Pi
1296 324 216 324 1296

Q.14 12/13 Q.15 11/50 Q.16 27/83

4 7
Q.17 (i) 1/8 (ii) 1/2 (iii) 47/64 Q.18 , Q.19 (a) 5/21 (b) 4/21 (c) 4/7
11 11
Q.20 64/189

EXERCISE-5
189 5
Q.1 130 Q.2 Q.3
256 8

64 24 5
Q.4 (a) ; (b) Q.5 P H / S = 1/2 Q.6
455 91 16

98
Q.7 n+1 : n1 Q.8 16 Q.9
243
Q.10 317

BANSAL CLASSES Private Ltd. ‘Bansal Tower’, A-10, Road No.-1, I.P.I.A., Kota-05 Page # 67
in Last Nineteen Years

You might also like